Sunteți pe pagina 1din 364

INFECTOLOGIA

1RA VUELTA
1RA CLASE
26/SET/2016
DR. EDUARDO MATOS PRADO
MEDICO INFECTOLOGO
SERVICIO DE INFECTOLOGIA - HOSPITAL NACIONAL ARZOBISPO LOAYZA

BACTERIOLOGIA

MICROORGANISMOS

ACELULARES

CELULARES

PROCARIOTAS

VIRUS: cidos nuclicos y


protenas
VIROIDES: molculas de RNA
PRIONES: protenas infecciosas

bacterias, algas
cianofceas

EUCARIOTAS

protozoos, hongos y
algas microscpicas.

LA PARED CELULAR EN EN PROCARIOTAS


FUNCIN:
Envoltura rgida que
proporciona proteccin
frente a choques
osmticos.
TIPOS DE PARED:

COMPOSICIN QUMICA:
Peptidoglucano o murena, formado por N-acetilglucosamina,
cido N-acetilmurmico y un tetrapptido. Exclusivo de
Bacteria.

GRAM POSITIVAS (retienen


el cristal violeta)
cido
lipoteicoico
PEPTIDOGLUCANO
(90%)

GRAM NEGATIVAS(no
retienen el cristal violeta)

cido teicoico

LPS
Porina

Lpido A

Membrana
plasmtica

Membrana plasmtica

PEPTIDOGLUCANO
(10%)

TINCIN BACTERIANA

TINCIN GRAM
Bacteria Gram +

Bacteria Gram -

Pared celular simple

Pared celular compleja

Capa de
peptidoglucano gruesa

Capa de
peptidoglucano fina

No capa externa de
lipopolisacridos

Capa externa de
lipopolisacridos

Retienen cristal
violeta/iodo-color
azul/violeta

Retienen FUCSINA o
SAFRANINA
rojo/rosado

Streptococcus de importancia mdica


Especie

Grupo de
Lancefield

Habitat

Infeccin en humanos

S. pyogenes

Humanos

Faringitis, Glomerulonefritis
(ASLO)

S. agalactiae

Bovinos. Humanos

Meningitis neonatal, Sepsis,


Infecciones ginecolgicas

S. equisimilis

Humanos y animales

Endocarditis, Bacteremia,
neumona, IRA superior

E. faecalis
S. bovis

S. viridans

Infeccin intrahospitalaria
D

Tracto gastrointestinal
humano y animales,
productos lacteos
S. mutans, S. mucosa oral,
salivarius, S. respiratoria y
sanguis, S.
gastrointestinal de los
mitis y S.
mamferos y del tracto
anginosus
genital en la
mujer

Infeccin del tracto biliar o


urinario, endocarditis, cncer
de colon
Caries dental, bacteriemia,
endocarditis infecciosa sobre
vlvula protsica
de comienzo tardo

Infecciones por Anaerobios:


Patgenos Anaerobios ms importantes del
Tubo Digestivo
La mayor concentracion de bacterias anaerobias se hallan en el leon y colon: constituyen el
99.9%.
Aproximadamente 400 especies microbianas.
Especies de Peptostreptococcus
Especies de Fusobacterium
Especies de Bacteroides: (principalmente el B. fragilis)

1.

Correlacione:

1. Estreptococo del grupo A


2. Estreptococo mutans
3. Estreptococo bovis
4. Estreptococo pyogenes
( ) Hemolisis beta y ASLO(+)
( ) Pertenece al grupo viridans
( ) Asociado a cncer de colon
( ) Estreptodornasa

A. 2,1,4,3
B. 4,3,1,2
C. 3,2,4,1
D. 2,4,3,1
E. 1,2,3,4

CLAVE: E

2. El componente patgeno MS importante que comparten el S.


pneumoniae, H. influenzae y la N. meningitidis, en una meningitis
bacteriana es:
A. Protena A
B. Capsula
C. Endotoxina
D. b-lactamasa
E. Catalasa(+)

CLAVE: B

3. Marque lo correcto con relacin a las clamidias y Rickettsias :


A. Estar dispersas por picadura de artrpodos infectados
B. Resistentes a los antibiticos usuales de amplio espectro
C. Contienen ADN o ARN pero no ambos
D. Son intracelulares obligados
E. Se reproducen solo por fisin binaria

CLAVE: D

4. Cul es la propiedad compartida por la Legionella pneumophila,


Mycobacterium avium y un Mycobacterium atpico:
A. Los cidos teicoicos son responsables de su antigenecidad
B. Acido alcohol resistente
C. Requiere hierro y cistena para su crecimiento
D. No es fcil la trasmisin de persona a persona
E. Son incapaces de crecer en medios de cultivo de laboratorio

CLAVE: D

PARASITOLOGIA

CLASIFICACIN DE PARSITOS
I.-Protozoarios:
Rizpodos: A. Hystoltica.
Flagelados: G. lamblia.
Coccidia: Isospora, Criptosporidium, Ciclospora.
II.-Helmintos:
Nemtodes: Ascaris, Uncinarias, Enterobious,
Strongiloides, Trichuris.
Cstodes: T-Saginata, Diphilobotrium, H Nana.
Tremtodos: Fasciola heptica.

18

PROTOZOOS

19

GIARDIASIS
Es la infeccin del intestino delgado por el protozoario
flagelado Giardia lamblia o intestinalis
trofozoito: tiene forma de gota
quiste : se desarrollan cuando las heces liquidas se
deshidratan gradualmente en su trnsito por el colon
GIARDIASIS AGUDA
Periodo de incubacin: 3-20 das (7 das)
Diarrea liquida, esteatorrea, nausea, balonamiento y
disconfort abdominal, flatulencia
Anemia por malabsorcion
Infeccin parasitaria intestinal mas frecuente
DIAGNOSTICO
Examen directo de heces
Examen de la cuerda encapsulada (ENTEROTEST)
TRATAMIENTO
Metronidazol 250mg 3v/d x 5 das o tinidazol
20

AMEBIASIS
Enfermedad parasitaria potencialmente mortal, producida
por Entamoeba histolitica
El hombre es el nico hospedero.
La transmisin es por va oral, al ingerir agua o alimentos
contaminados
Diarrea disenterica, absceso hepatico amibiano
Tratamiento
Intraluminal
Diiodohidroxiquinoleina
Colitis invasiva Metronidazol 750 mg vo TID por 7 a 10 das
Colitis invasiva no severa Tetraciclina 250 mg vo TID x10 das
Absceso Heptico Igual a colitis invasiva

21

BALANTIDIOSIS
Es un protozoario ciliado; el mas grande que
parasita al hombre
Habitat: Intestino grueso del cerdo, hombre y
mono.
La forma infectante para el hombre: quiste
Puerta de entrada: Via oral
Mecanismo de transmisin: Agua y alimento
contaminado
Antecedente de pacientes: contacto con cerdos
Disentera balantidiana se caracteriza por
Diarrea con moco y sangre, Pujo y tenesmo
Tratamiento
Tetraciclina
Nios : Metronidazol

Diagnostico:

BLASTOCISTOSIS
Blastocystis hominis. Habita en el
intestino del hombre y de otros animales
(monos, simios, cerdos, conejos, caballos,
etc.).
transmisin es fecal oral
poseen seudpodos para su locomocin
y alimentacin
Carece de pared celular, pero contiene
mitocondria, aparato de Golgi y retculo
endoplsmico . Su reproduccin es
usualmente por fisin binaria y crece en
cultivos bajo condiciones anaerbicas.
Pacientes
con
algn
grado
de
inmunosupresin, pero aun no se
considera como oportunista

Seriado de Heces y Cultivos


Tcnicas inmunolgicas:
inmunofluorescencia en heces

NEMATODES

24

ENTEROBIOUS VERMICULARIS
Tiene un ciclo de vida muy simple
El parasito adulto vive en el tracto GI , principalmente en el
ciego y apndice
Las hembras depositan sus huevos en la piel perianal. Esto
ocurre usualmente en la noche
Prurito anal; reaccin inflamatoria a la presencia del
parasito adulto y los huevos. Ocurre predominante de
noche
Sntomas abdominales dolor abdominal, nausea y vmitos
Enterocolitis
eosinofilicas,
incluso
apendicitis,
Vulvovaginitis, salpingitis, ooforitis, granulomas cervical o
inflamacin peritoneal
Tambin se ha descrito infestacion de mucosa nasal
Diagnostico: La prueba de la cinta scotch o test de Graham
TRATAMIENTO
Albendazol 400 mg una sola vez

25

ASCARIS LUMBRICOIDES
MANIFESTACIONES PULMONARES Y DE
HIPERSENSIBILIDAD
Transmision por ingesta de huevos maduros
(demoran 3 4 semanas en madurar en el suelo)
Realiza ciclo de Loss
El sndrome de Loeffler o neumonitis suele
ocurrir una o dos semanas despus de la
ingestin de los huevos.
La urticaria relacionados a la hipersensibilidad
ocurre hacia el final del periodo de migracin a
travs de los pulmones
Tratamiento

Albendazo
Piperazina (Obs. Intestinal),

26

TRICHURIS TRICHIURA
o tricocefalosis
La transmisin de la Tt ocurre va fecal-oral
Los huevos ingeridos, alcanzan el intestino
para liberar la larva
El macho y la hembra viven en el colon
ascendente y el ciego
Las heces son blandas y con frecuencia
contienen moco y/o sangre. Defecacin
nocturna es muy comn
El prolapso rectal es el signo clnico mas
caracterstico
Diagnostico: Se hace por examen de heces
para la investigacin de huevos
Los pacientes pueden tener una eosinofilia de
hasta el 15%
Tratamiento: albendazol x 3 dias

27

UNCINARIAS
(Necator americanus, Ancylostoma duodenale)
Penetra por piel.
De la piel la larva pasa a los pulmones.
8 a 21 das despus de la infeccin, la larva, cruza la
vasculatura pulmonar y entra al sistema respiratorio,
asciende por el rbol traqueo bronquial hasta la
faringe, y entonces es deglutido. (Ciclo de Loos).
sndrome de Loeffler. Eosinofilia
En el ID, la larva madura a parasito adulto
Los parsitos se prenden de la mucosa para
alimentarse de sangre y protenas
Produce anemia por micro hemorragias
Diagnostico: Examen de heces para huevos o parsitos
maduros
Tratamiento:
Albendazol x 3 dias
28

STRONGILOIDES STERCORALIS
1. Invasion de la piel
2. Migracion de la larva

3. Penetracion de la mucosa
intestinal
Sindrome de hiperinfestacion

Asociacion con HTLV -1

Tratamiento
Ivermectina
Thiabendazol

29

CESTODES

30

TENIASIS (intestino delgado)


TENIA SAGINATA
Escolex con 4 ventosas, sin rostelos ni
ganchos.
Proglotide inmaduro, maduro, grvido (12-30
ramas, contiene hasta 100,000 huevos)
El ganado vacuno se infecta al ingerir pastos
contaminados.
Los embriones u oncosferas liberadas
penetran la mucosa intestinal y entran en la
circulacin y son transportados por todos el
cuerpo
Fase larvaria (Cisticerco bovis) en msculos de
bvidos
Adulto: 4-10 metros de largo, compuesta de
1,000-2,000 proglotidos
Hospedero intermediario (vacuno)
Longevidad: >25 aos

TENIA SOLIUM
Escolex con 4 ventosas
Un rostelo con doble corona de ganchos
Proglotide grvido(< de 12 ramas) contienen entre 30,000 y 50,000 huevos
Huevo similar al de T. Saginata
Fase larvaria; Cysticercus celulosae
Hospedero intermediario: cerdo
Longevidad > 25 aos
Longitud: 5-7 metros

CISTICERCOSIS
La cisticercosis es una parasitosis de
humanos y cerdos causada por el
metacestodo de Taenia solium, el cual
presenta el esclex del parsito adulto,
invaginado en una vescula.
La infeccin se adquiere al ingerir
huevos o progltidos grvidos del
cestodo, eliminados con las heces del ser
humano infectado con Taenia solium.
La invasin a sistema nervioso central
(SNC) da lugar a la neurocisticercosis.
Es la principal causa de convulsiones en
adultos.
32

HYMENOLEPIASIS
Los agentes etiolgicos son Hymenolepis nana e Hymenolepis
diminuta.
H. nana es el nico cestodo cuyo ciclo biolgico no requiere
de hospederos intermediarios.

Hymenolepis diminuta, parsito de roedores,


por la ingesta accidental de artrpodos hospederos
intermediarios infectados con cisticercoides,
habitualmente en cereales, harinas, especies, fruta
seca, alimentos para mascotas.

Se realiza mediante estudios coproparasitoscpicos en


fresco, de concentracin
Tratamiento
Praziquantrel. Destruye a parsitos adultos y
cisticercoides. Dosis nica: 25 mg/kg

33

DIFILOBOTRIASIS
D. latum, D. pacificum
El gusano adulto es el gusano plano ms
largo (mide hasta 25m): se adhiere al ileo
El gusano adulto tiene 3000-4000
progltides, los cuales liberan 1 milln de
huevos diariamente en las heces.
Puede desarrollar deficiencia de Vit B12
Hasta 2% de pacientes infectados tienen
anemia megaloblstica.
Deteccion de huevos en las heces
Puede detectarse eosinofilia leve a
moderada
PRAZIQUANTEL ES ALTAMENTE EFECTIVA:

5-10 MG/KG UNA DOSIS

34

HIDATIDOSIS
Hidatidosis infeccin en los seres humanos por la
fase larvaria del Echinococcus granulosus, E.
multilocularis, o E. vogeli.
Hospedadores definitivos: caninos que eliminan
los huevos en sus heces.
Despus de la ingestin de huevos, se desarrollan
quistes en hosp. intermediarios -ovejas, vacas,
seres humanos, cabras.
Cuando un perro (E. granulosus) ingiere carne
que contenga quistes infectados, el ciclo de vida
se ha completado
Rx de Trax e imgenes TAC, TEM, RMN, definir
quistes de E. granulosus en el pulmn
Serodiagnostico ELISA, Western Blot), un
resultado negativo no excluye el diagnstico de la
equinococosis.
TRATAMIENTO: Albendazol, por 4 sem.

35

TREMATODES

36

PARAGONIMOSIS
Zoonosis parasitaria
Invasion de larvas de trematodes Gnero Paragonimus a pulmon,
TCSC, cerebro.
No es contagiosa persona a persona.
Los hospederos intermediarios son dos: un caracol y un cangrejo
de agua dulce.
El hombre se infecta al ingerir cangrejos insuficientemente
cocidos que contiene las formas infectantes, las metacercarias.
MIGRACION:
Llega a estomago o intestino, se desenquista
Penetra pared abdominal, llega a cavidad peritoneal,
atraviesa diafragma y pleura parietal
Puede ser llevada porcirculacion linfohematica a cerebro,
pericardio, TCSC y organos genitales.
Pulmonar: Esputo hemoptoico o caf, Fiebre, Dolor toracico
pleurtico

Praziquantel a la dosis de 25 a 30 mg/Kg de peso


durante 2 - 4 das .

FASCIOLOSIS
1.
Aguda o invasiva que
corresponde a la migracin de la
forma juvenil del parsito por el
hgado (2 - 3 meses). Eosinofilia
2.
Crnica o de estado que
corresponde a la ubicacin del
parsito en las vas biliares.
3.
Portador
Triclabendazol
Dosis: 10-12 mg/kg 2 dosis
interdiario por 2 dias

38

5. De los siguientes parsitos, Cul produce anemia y diarrea con ms


frecuencia?:
A. Oxiuro.
B. scaris.
C. Tricocfalo dispar.
D. Giardia lamblia.
E. Hymenolepis nana.

CLAVE: D

6.

Respecto a las parasitosis, son verdaderas:

1. scaris lumbricoides es un nematodo


2. La teniasis tiene como habitat el intestino delgado
3. La maduracin del huevo del scaris dura 3 semanas en condiciones adecuadas
4. Giardia lamblia es un trematodo
5. Tinidazol se indica para Giardasis
A. 1,2,3,4
B. 1,2,3,5
C. 2,3,4,5
D. 1,2,3,5
E. 1,2,5

CLAVE: D

7. Uno de los siguientes estadios de Taenia solium atraviesa la pared del


intestino del hombre y se disemina en los tejidos:
A. Proglotide
B. Cisticerco
C. Embrin
D. Esclex
E. Estrbilo

CLAVE: C

8. Cules de los siguientes parsitos producen sndrome de Loeffler?:


1-Oxiuros,
2-Tenias,
3-Uncinarias,
4-Giardias,
5-scaris,
6-Criptosporidium
A. 2,6
B. 2,3
C. 3,5
D. 4,5
E. 1,3

CLAVE: C

MICOLOGIA

MICOSIS SISTMICAS DE IMPORTANCIA CLNICA


1.

Levaduras:
Candidiasis : C. albicans
: C. no albicans
Criptococosis:
: C. neoformans
2.
Filamentosos:
Aspergilosis
Zigomicosis (mucorales)
Fusariosis (Fusarium)
3.

Dimrficos (endmicos):
Histoplasmosis (Histoplasma)
Paracoccidioidomicosis (Paracoccidioides brasiliensis)

4.

Oportunistas:
Pneumocystis jiroveci

Micosis
Superficiales
Dermatomicosis
Piedra negra
Piedra Blanca
Pitiriasis versicolor

Dermatofitosis
Tias

De implantacin o
Subcutneas
Esporotricosis
Cromomicosis
Micetoma
Lobomicosis
Rinosporidiomicosi
s
Basidiomicosis

Micosis
Sistmicas

Histoplasmosis
Coccidioidomicosis
Paracoccidioidomicosis
Blastomicosis

Oportunistas

Candidosis
Criptococcosis
Mucormicosis
Aspergilosis
Neumocistosis

ASPERGILOSIS
Aspergillus, un hongo ubcuo
Infecciones ocurren en :
19% pacientes con leucemia
20-30% pacientes
Inmunocomprometidos
Los patgenos ms comunes
Aspergillus fumigatus
Aspergillus flavus
Aspergillus fumigatus involucrado en 80
% de casos
Manifestaciones clinicas
Aspergilosis alrgica
Aspergilomas(bola fngica)
Aspergilosis Invasiva
- compromiso pulmonar en 90-95 %
- diseminacin extrapulmonar en 25 %

TRATAMIENTO
Anfotericina B 1.0mg/kg/dia
Itraconazole 200-400mg.

CRIPTOCOCOSIS
AGENTE CAUSAL:
Criptococo Neoformans
DIAGNOSTICO
Lquido cefaloraqudeo
Tinta china 60 80%
Cultivo en medio de Sabouraud 90%
Test de ltex cerca al 100%
Tratamiento:
Anfotericina B + fluocitosina por 2 sem.
luego fluconazol por 3 meses

HALLAZGOS TPICOS EN EL LCR


Presin de apertura
incrementada (>18cm H2O en >
60% de pacientes)
Clulas blancas, >20, usualmente
linfocitos
(>60% de pacientes)
Proteinas >40%mg/100ml (>50%
de pacientes)
Glucosa normal
Ttulos de AgCr positivo y menor
que el AgCr de suero (91% de
pacientes)

9. La prueba de la tinta china es til pare diagnosticar una infeccin por:


A. Aspergillus niger.
B. Cryptococcus neoformans.
C. Rhizopus.
D. Cndida tropicalis.
E. Histoplasma capsulatum

CLAVE: B

10. Hongo que causa neumona en inmunodeprimidos es y la levadura


adquirida por inhalacin que causa meningitis es:
A. Histoplasma capsulatum/Coccidioides
B. Cndida albicans/Rhizosporum nigricans
C. Aspergillus fumigatus/Criptococo neoformans
D. Sporothrix schenckii/ Malassezia furfur
E. Pneumocystis jirovecii/Blastocystis

CLAVE: C

11. Cul de los siguientes hechos es preciso para el diagnstico definitivo


de infeccin por Aspergillus?
A. Elevacin al cudruple de los ttulos sricos en el 2do control
B. Aislamiento en cultivos de esputo
C. Demostracin de invasin tisular
D. Demostracin de material gentico en muestras tisulares mediante PCR
E. Visualizacin del hongo mediante tinta china

CLAVE: C

VIROLOGIA

VIRUS: CONCEPTO

Son parsitos intracelulares obligados.


Son microorganismos de pequeo tamao (agentes filtrables).
Tienen mecanismos especiales de replicacin.
Material gentico (ADN o ARN) capaz de replicarse de forma autnoma.
Capa proteica o cpside que rodea al material gentico
A veces una envuelta membranosa cubriendo a la capa proteica (virus envueltos y desnudos).

VIRUS DNA

VIRUS RNA

POXVIRUS Viruela, Molusco


HERPESVIRUS
Varicela-Zoster, Herpes Simple,
VEB, CMV
ADENOVIRUS Conjuntivitis
HEPADNAVIRUS Hepatitis B
PAPOVAVIRUS Papiloma virus
humano

RABDOVIRUS Rabia
PARAMIXOVIRUS Sarampin
PICORNAVIRUS Poliovirus
RETROVIRUS VIH, HTLV I - II
FLAVIVIRUS Fiebre Amarilla
ORTOMIXOVIRUS Influenza

54

Virus caractersticas y
clasificacion

CICLO DE MULTIPLICACION VIRAL

1. Adsorcin de los virus a las

m. plasmticas.
2. Penetracin

3. Decapsidacin
4. Transcripcin
5. Traduccin
6. Replicacin.
7. Ensamblaje
8. Liberacin
56

HSV II

HERPES VIRUS

HSV I

Gingivoestomatitis, faringitis
P. Incubacion: 2 14 das
Fiebre, dolor faringeo, eritema
Vesiculas en mucosa faringea y oral
Adenomegalias cervicales
Duracion: 10 14 das. No secuelas
Encefalitis herptica: lobulos
temporales.

Infeccion genital primaria (70 95%)


Mas leve en coinfeccion herptica
P. Incubacion: 2 7 das.
Localizacin: glande, cuerpo de pene, vulva perin,
vagina, cuello uterino.
Lesiones extrainguinales: 10 20% de casos.
Fiebre, anorexia, adenomegalia.
Recurrencias en mas 90% antes del ao y tan
intensas como la primoinfeccin sobre todo en
mujeres

TRATAMIENTO
Aciclovir valaciclovir

VIRUS VARICELA
ZOSTER
Produce 02 enf. distintas:
varicela y herpes zoster.
Seres humanos unico reservorio
Diseminacion por va area.
Ataque secundario: 70 90%
Infectante: 48horas antes de
aparicion de vesiculas , hasta
aparicion de costras.
Latente en ganglios de raices
dorsales.
4% puede sufrir segundo
episodio de HZ
HZ en nios sin varicela previa.

Varicela:

Nios de 2 12 aos. Tto no


recomendado.
Adolescentes y adultos
jovenes: Aciclovir 800 mg.
5v/d x 5-7 das.

Herpes zoster.

Huesped
inmunocompetente:Aciclovir 800
mg. 5v/d x 7 10 das.
58

VIRUS DE LA HEPATITIS B
Un hepadnavirus1
Los humanos son los nicos
huspedes2
Genoma de DNA de doble hlice
parcial1,2
Core:1,2

Virus de hepatitis B

Pre-S
Ags
DNA

Antgeno c hepatitis B (HBcAg)


Antgeno e hepatitis B (HBeAg)

Envoltura:1
antgeno de superficie de la
hepatitis B (HBsAg)
antgeno pre-S (pre-S Ags)

HBsAg
HBeAg
(HBcAg)

1. WHO, WHO/CS/CSR/LYO/2002.2: Hepatitis B


2. 2. CDC, Hepatitis B, Pink book, 2007

CARACTERISTICAS ESPECIALES DEL VHB


Es 80 veces ms contagioso que VIH

El volumen de sangre requerido para


adquirir la infeccin es de 0,00004 ml.

El virus de la hepatitis B puede


sobrevivir fuera del organismo por lo
menos siete das

El periodo medio de incubacin de la


hepatitis B es de 75 das, pero puede oscilar
entre 30 y 180 das

Hepatitis crnica B : curso serolgico tpico

12. Marque lo correcto con relacin a los virus:


A. Los virus que tienen como genoma ARN tienen un ciclo de replicacin
ms complejo que los de ADN
B. Los virus ncleo citoplasmticos de ADN infectan a otros virus
C. Es indispensable la sntesis de ARNm para la duplicacin de los virus
D. Los virus que tienen genoma ADN se sintetizan en el citoplasma del
husped
E. Su ciclo reproductivo comprende: adsorcin, adhesin, ensamblaje,
replicacin, y lisis

CLAVE: C

13. Marque V o F segn corresponda:


( ) El periodo de incubacin de VHS vara entre 2-14 das
( ) El herpes genital por VHS-2 en pacientes infectados por VHS-1 es ms leve
( ) La primoinfeccin es ms leve que las recurrencias
( ) Los pacientes con VHS-2 recurren un 90% en el primer ao
( ) Es fcil diferenciar una faringitis herptica de una bacteriana
( ) La recidiva del VHS-2 es aproximadamente 8 veces ms que del VHS-1 en
genitales
( ) En la estomatitis herptica se respetan las encas
A. VVFVFVF
B. VVFFVVF
C. FFVVFFV
D. FVFVFVF
E. VFVFVFV

CLAVE: A

14. Correlacione las propiedades siguientes, con los virus que le correspondan:
1. Genoma segmentado
2. Cuerpo de inclusin intranuclear
3. Cuerpos de inclusin intracitoplasmticos
4. Meningitis asptica
5. Muy resistente a la activacin por agentes qumicos y fsicos
a. Echovirus
b. Virus Influenza
c. Citomegalovirus
d. Virus Rabia
e. Virus Hepatitis B
A. 1B,2C,3A,4D,5E
B. 1A,2B,3D,4C,5E
C. 1A,2C,3C,4E,5A
D. 1B,2C,3D,4A,5E
E. 1C,2D,3A,4E,5B

CLAVE: D

ANTIBACTERIANOS

PENICILINAS

PENICILINAS

NATURALES

PENICILINASA
RESISTENTE

SEMI SINTETICAS

PENICILINA G

METICILINA

PENICILINA V

OXACILINA

AMOXICILINA
AMPICILINA

DICLOXACILINA
ANTI SEUDOMONAS

TICARCILINA
CARBENICILINA

MEZLOCILINA
PIPERACILINA

PENICILINA G Y V

ESPECTRO DE ACCIN:
PENICILINA G y V:
Por va IV: penicilina
cristalina o acuosa.
Por va IM o depsito:
penicilina procanica
(con anestesico) y
benzatnica
Penicilina procanica:
V1/2 1 - 3 horas
Penicilina benzatnica:
va IM, absorcin lenta,
efectos por 21 - 26 das

Cocos gram + aerobios: S. viridans,


agalactiae y pyogenes, neumococo
T. pallidum y leptospira
C. diftheriae y L. monocytogenes
Actinomicosis
Listeria
Peptostreptococcus, Clostridium
perfringens, C. tetani
NO: enterococos, estafilococos y
gonococos

AMINOPENCILINAS o PENICILINAS
SEMISINTTICAS: amoxicilina , ampicilina
Sustitucin Amino en la cadena lateral.
No son resistentes a las
betalactamasas.
Se prefiere el uso de la amoxicilina por:
Mejor absorcin
Tener mejores niveles plasmticos.
menores efectos gastrointestinales.

Listeria monocytogenes, Strep.


Viridans, enterococo.

PENICILINAS ANTIESTAFILOCOCICAS:
Meticilinas
Meticilina, oxacilina y dicloxacilina
Grmenes sensibles: meticilino sensibles
Cubre contra infecciones por estafilococo aureus y epidermidis
productores de penicilinasas

MRSA

PENICILINAS ANTIPSEUDOMONA

Carboxipenicilinas
Ticarcilina:
activa contra pseudomona.

Ureidopenicilinas
piperacilina.
activa contra pseudomona
Ineficaces contra MRSA

CEFALOSPORINAS

CEFALOSPORINAS 1RA GEN.


Actividad frente a estreptococos y estafilococos, pero no a
enterococos
Utiles en profilaxis de Infeccin de herida operatoria.
tiles en infecciones de piel y tejidos blandos.
Alternativa para alrgicos a penicilina
VIA ORAL
CEFADROXIL
CEFALEXINA
CEFRADRINA
VIA PARENTERAL
CEFAZOLINA
CEFALOTINA

CEFALOSPORINAS 2DA GEN.


Util en infecciones por M. Catarrhalis, N. Meningitidis y N.
gonorrheae
Contra S. Pneumoniae y S. Pyogenes es mas activa que Cefalo 1ra.
pero menos para S. aureus
Utiles en infecciones respiratorias extrahospitalarias.
Mayor accion contra Gram negativos
2da generacion :
CEFACLOR
CEFUROXIMA AXETIL
CEFUROXIMA
CEFOXITINA

CEFALOSPORINAS 3ra. Gen.

1.
2.
3.
4.

Marcada actividad frente a gram negativos


Gran actividad frente a enterobacterias y H. Influenza.
Menor actividad que generaciones precedentes a S.
aureus.
No actividad contra MRSA
Concentracin adecuada en LCR
Cefotaxima
Ceftriaxona
Ceftazidima
antipseudomonas
Cefoperazona

CEFALOSPORINAS de 4ta Gen.


Espectro ampliado a gram (+) y gram (-)
No es mas activo ante pseudomona que ceftazidima.

CEFEPIME

Cefalosporina de 4ta generacin


Excretado por via renal
Estable a la hidrlisis de -lactamasas
Pobre inductor de -lactamasas de tipo I

Cefalosporinas de 5ta gen.


Ceftobiprol
Ceftarolina

Afinidad por PBP2a y PBP2x.

Staphylococcus aureus como estafilococos coagulasa negativos, sensible y


resistente a meticillina, as como frente a las cepas resistentes a vancomicina
(VRSA).

GLICOPEPTIDOS
Se unen al dipptido final Dalanil-D-alanina impidiendo su
transferencia a la murena
Bactericidas
Uso restringido a infecciones
hospitalarias y/o severas
resistentes por Gram (+)
No activo contra gram negativos
ni anaerobios

Debe estar restringida al tratamiento


de:
Infecciones de G + resistentes a
betalactamicos
Pacientes con hipersensibilidad a
betalactamicos
Colitis asociada a ATB con riesgo de
vida que
no responden a Metronidazol

VANCOMICINA: Farmacocintica
VO: poca absorcin
excretado por heces en gran
cantidad
No IM
90% se excreta por
FILTRACION GLOMERULAR
Ajustar la dosis en individuos
con Insuficiencia Renal:
- Depuracin creatinina
- Hemodializable

EFECTOS ADVERSOS
OTOTOXICIDAD.
NEFROTOXICIDAD.
Hipersensibilidad
Flebitis y dolor en el sitio de inyeccin IV,
A veces: escalofros, erupciones y fiebre
Goteo IV rpido:
Reacciones eritematosas o urticarianas
Hiperemia Rubor CUELLO ROJO y HOMBRE
ROJO por degranulacion de mastocitos

Taquicardia
Hipotensin

QUINOLONAS
Mecanismo de Accin:

Quinolonas de
Primera
Generacin

Quinolonas de Segunda Generacin

cido nalidxico

Subgrupo1:

Actividad
bacteriolgica:
Enterobacterias

Penetracin:
Slo urinaria

Norfloxacina
Actividad bacteriolgica:
Enterobacterias
Ps. aeruginosa

Penetracin:

Slo urinaria

Subgrupo 2
Ofloxacina
Ciprofloxacina
Actividad bacteriolgica:
Enterobacterias
Ps. aeruginosa
Grmenes atpicos:
Chlamydia,
Mycoplasma, Legionella
Penetracin:
Sistmica (intracelular)
Urinaria

Quinolonas de Tercera
Generacin

Quinolonas de Cuarta
Generacin

Levofloxacina

Moxifloxacina

Actividad bacteriolgica:

Actividad bacteriolgica:

Enterobacterias
Ps. aeruginosa (+/-)
Grmenes atpicos
Estreptococos
Estafilococo sensible a
meticilina

Penetracin:
Sistmica (intracelular)
Urinaria

Enterobacterias
Ps aeruginosa (+/-)
Grmenes atpicos
Estreptococos
Estafilococo sensible a
meticilina
Anaerobios

Penetracin:
Sistmica (intracelular)
Urinaria

ANTIBIOTICOS QUE ACTUAN EN LA SNTESIS DE


PROTENAS
Subunidad 30S:
Aminoglicsidos:

Tetraciclinas.
Glicilciclinas

Subunidad 50S:
Oxazolidinonas
Lincosamidas/cloranfenicol.

Macrlidos/Cetlidos

AMINOGLUCOSIDOS
Antibiticos bactericidas actan 30S
Activos principalmente contra Gram
negativos aerobios
Su poder bactericida radica el pico
srico que alcanza

Estreptomicina
Kanamicina
Gentamicina
Tobramicina
Amikacina

No actuan a Ph cido (secreciones


bronquiales), ni en tejidos de baja
oxigenacin.
Es preferible dar en dosis nica mantiene
su efectividad y disminuye el riesgo de
toxicidad.
Sinergismo con los B lactmicos
REACCIONES ADVERSAS
1. Nefrotoxicidad (5-25%):
2. Ototoxicidad (2-4%):

Es irreversible.

Gentamicina (mas vestibular) y


amikacina (mas dao auditivo)

LINCOSAMIDAS

CLINDAMICINA
Se introdujo como antiestafilococo.
Posteriormente antianaerobio .
Riesgo de colitis por Clostridium difficile ha
limitado su uso
Acta inhibiendo la sntesis proteica
bacteriana - subunidad 50S
Inhiben la produccin de toxinas
estafilocccicas asociadas al sindrome de
shock txico y previenen la produccin de
biofilms

ESPECTRO ANTIBACTERIANO
Protozoarios: Toxoplasma gondii, Plasmodium y
especies de Babesia.
Otros microorganismos:
Tambin presenta alguna actividad contra
P.jiroveci, Leptospira spp. y Chlamydia spp.
REACCIONES ADVERSAS
Las ms comunes son: diarrea
Ms temible es la colitis seudomembranosa,
producida por C. difficile, que puede ser mortal.

MACROLIDOS
roxitromicina, claritromicina y
azitromicina son derivados semisintticos
de la eritromicina
Eritromicina y claritromicina poseen 14
carbonos en el anillo lactona
azitromicina es una mlecula de 15
carbonos, tambin conocida como
azlido
Innhiben la sntesis proteica subunidad
50S
Se concentran dentro de macrfagos y
polimorfonucleares, patgenos
intracelulares.
No se recomiendan para infecciones
bacterimicas por sus escasos niveles
en sangre.

Gran actividad:
Mycoplasma
Chlamydia
Campylobacter
Toxoplasma
Legionella
Eventos adversos
G-I: dolor epigstrico intenso, vmitos,
diarrea (estimulan motilina)
Hepticas: hepatitis Colestasica (luego 10-20
das)
Vrtigos, acfenos y sordera en dosis altas
(reversible)
SN: confusin, paranoia, alucinaciones miedo,
pesadillas

15. Cul de entre los siguientes, Ud. escogera como antibitico, si su


paciente es resistente a la Oxacilina.. y si es resistente a la
Gentamicina:
A. Ceftriaxona/Tobramicina
B. Dicloxacilina/Estreptomicina
C. Vancomicina/Amikacina
D. Teicoplanina/ Estreptomicina
E. Azitromicina/ Neomicina

CLAVE: C

16. Cul es el efecto adverso ms frecuente en la administracin de


Clindamicina?:
A. Colitis pseudomembranosa.
B. Nefrotoxicidad.
C. Ototoxicidad.
D. Anemia aplsica.
E. Artropata.

CLAVE: A

17. El sndrome de hombre rojo es un efecto adverso de la Vancomicina,


que se presenta por...
A. Estimular poliglobulia.
B. Generar taquicardia.
C. Estimulacin de centros pirgenos.
D. Liberacin de histamina por mastocitos.
E. Extravasacin de hemates.

CLAVE: D

18. Cul de las siguientes series de Cefalosporinas corresponde a un


ordenamiento de mayor a menor espectro antibacteriano?:
A. Cefpirome, cefaclor, ceftriaxona, cefalotina
B. Cefpirome, ceftriaxona, cefaclor, cefalotina
C. Ceftriaxona, cefaclor, cefalotina, cefpirome
D. Ceftriaxona, cefalotina, cefaclor, cefpirome
E. Cefalotina, cefaclor, ceftriaxona, cefpirome

CLAVE: B

19. Marque lo correcto con relacin a las penicilinas:


A. La Penicilina G sdica se da solo cada 6 horas.
B. La Penicilina G procanica tiene un anestsico asociado.
C. La penicilina G potsica se da nicamente en casos de meningitis muy
grave.
D. La penicilina benzatnica solo se da mximo 3 das.
E. La penicilina V tiene ms espectro que la penicilina G.

CLAVE: B

ANTIPARASITARIOS

BENZOIMIDAZOLES
ALBENDAZOL, MEBENDAZOL Y TIABENDAZOL

PIPERAZINA

PAMOATO DE PIRANTEL /OXANTEL

PRAZIQUANTREL

IVERMECTINA

NITROIMIDAZOLES: metronidazol, tinidazol, secnidazol

IVERMECTINA

CLOROQUINA

20. Cul de las siguientes afirmaciones sobre la Cloroquina es cierta:


A. Es activa frente a las formas extra hemticas del paludismo.
B. Es til para prevenir la infeccin paldica inicial, as como en las
recadas.
C. El tratamiento prolongado a dosis altas puede producir atrofia ptica.
D. Su lugar de accin principal son los canales de Ca+ en las membranas
celulares de Plasmodium.
E. Se usa frecuentemente en el tratamiento de las infecciones por filarias y
nematodos.

CLAVE: B

21. Cual efecto adverso ocurre con el uso de Mebendazol durante la terapia
por nematodos intestinales:
A. Ictericia colestsica
B. Opacidades corneales
C. Reaccin de Mazzotti
D. Neuropata perifrica
E. Ninguno de los anteriores

CLAVE: E

22. Uno de los frmacos usados para el tratamiento de la Triquinosis es el


Tiabendazol, su mecanismo de accin es:
A. Inhibicin del trasporte de glucosa
B. Inhibicin de la sntesis de ADN
C. Enlace a ferriprotoporfirina IX
D. Inhibicin de ADN girasa
E. Dao por radical libre a la membrana parasitaria

CLAVE: A

23. El mecanismo de accin del Praziquantel frente a los esquistosomas,


parece ser:
A. Alteracin de la utilizacin de Ca+ por el esquistosomas
B. Alteracin de la utilizacin de Na+ por el esquistosomas
C. Alteracin de la utilizacin de glucosa por el esquistosomas
D. Alteracin de la utilizacin de ATPasa por el esquistosomas
E. Activacin de enzimas lticas de los esquistosomas

CLAVE: A

ANTIMICOTICOS

Mecanismo de Accin de los Agentes


Antifngicos:

ncleo

Anlogos
Nuclesidos
Flucitosina

Ergosterol
Polienos (AmB, LFAB)
Azoles Fluconazol,Itraconazol
Voriconazol
Posaconazol
106

Anfotericina B desoxicolato (AMB-d):


Indicaciones:
Blastomycosis
Fusariosis
Coccidioidomycosis
meningitisCriptocococica (asociado a
flucitosina)
Moderada o severa aspergillosis
Histoplasmosis
Paracoccidioidomicosis Sporotricosis
Candidiasis

Profilaxis en
inmunodeprimido
TTO emprico neutropenia
febril

Desoxicolato
Complejos lipidicos

Liposomica
Dispersion coloide

Indicaciones

FLUCITOSINA:

Infeccion por Candida

Candidemia (asoc a Anfotericina)


Endoftalmitis Endocarditis

La flucitosina (5-fluorocitosina) es un
anlogo fluorado del nuclesido citosina.
El mecanismo de accin consiste en inhibir
la sntesis de DNA fngico.
Espectro de accin reducido: exclusiva
frente a hongos levaduriformes.
Raramente se usa sola, porque desarrollan
resistencia durante el tto.
El efecto adverso ms serio es la toxicidad
medular

Meningitis
Infeccion por Criptococo
Meningitis ( asoc a Anfotericina) *

TERBINAFINA
Mecanismo de accin: inhibe la
escualeno-epoxidasa, que cataliza
conversin de escualeno a escualeno-2,3
epoxido, precursor del lanosterol.

Farmacocintica:
biodisponibilidad oral 80%.
Lipoflica.
Altas concentraciones en tejido
adiposo y queratinizado.
Se metaboliza en hgado.
Vida 26 hs. Ajustar en disfuncin
renal o heptica.
Persiste en estrato crneo, pelo y
uas por hasta 12 semanas o ms.

Dosis: Oral: 1 vez al da. Tabletas de 250 mg.


Tpica: ungento 1% 2 veces al da
Usos clnicos: onicomicosis, tia capitis, otras
formas de tia. Otros: esporotricosis,
cromomicosis, eumicetoma, micosis endmicas.
Efectos adversos: nuseas, diarrea, dolor
abdominal, dispepsia, trastornos en piel,
colestasis.
Raros: neutropenia, trombocitopenia, NET,
angioedema. No interacciones.

AZOLES
Compuestos sintticos que presentan uno
o ms anillos imidazol
A partir de 1990 aparecieron los triazoles
(fluconazol, itraconazol,
voriconazol, posaconazol)
Mecanismo de accin: actan inhibiendo
la enzima 14 a demetilasa,
que impide la conversin del lanosterol
en ergosterol, principal esterol
de la membrana citoplasmtica.
Este mecanismo de accin es
fungisttico

EQUINOCANDINAS

Indicaciones
Infeccion por Candida

Actan inhibiendo no competitivamente la


enzima -1,3- D-glucan sintetasa la cual
interviene en la formacin de polmeros de la
pared celular fngica.

orofaringea esofagica
Infeccion por Aspergillus

Invasiva refractaria

RAMs
Molestias GI

Rubor
Transaminasemia

24. Un frmaco especialmente usado en neumonas por Pneumocystis


jirovecii es:
A. Paramomicina
B. Flumagilina
C. Ivermectina
D. Pentamidina
E. Nitazoxanida

CLAVE: D

25. Bloquea la desmetilacin del lanosterol al Ergosterol, interrumpiendo la


integridad de la membrana fngica:
A. Flucitosina
B. Griseofulvina
C. Anfotericina B
D. Ketoconazol
E. Icofungipen

CLAVE: D

26. Es metabolizado a un producto que inhibe el timilato sintetasa y evita la


sntesis de ADN fngico:
A. Flucitocina
B. Griseofulvina
C. Caspofungina
D. Anfotericina B
E. Ketoconazol

CLAVE: A

ANTIVIRALES

CULES SON LOS EFECTOS ADVERSOS DEL TARGA?

Herpes Simplex y V. Zoster


ACICLOVIR:
Derivado cclico de guanosina
Mecanismo de accin
Inhibicin sntesis DNA
Aciclovir timidina quinasa viral 200 veces + afn por aciclovir

Oral (15-20% biodisponibilidad) y E.V.


Reacciones adversas: trastornos GI, nefropata cristalina reversible

CITOMEGALOVIRUS
GANCICLOVIR:

Anlogo acclo de guanosina


Idem aciclovir, pero fosforilacin a ganciclo GMP catalizado por
quinasas virales y celularesganciclo GTP inhib DNA polimerasa
viral
Oral, e.v., implante intraocular
Retinitis por CMV y profilaxis (oral)
Mielosupresin: neutropenia 20-40% de los pctes

27. Cul de los siguientes frmacos usados en la terapia del SIDA


produce depresin de la mdula sea?:
A. Lamivudina
B. Didanosina.
C. Zalcitabina.
D. Estavudina.
E. Zidovudina.

CLAVE: E

28. Con relacin a los efectos adversos de los frmacos, indique cual de los
siguientes es el que puede causar nefrolitiasis. Y hepatitis severa
A. Nevirapina/Estavudina
B. Didanosina/lamivudina
C. Estavudina/Indinavir
D. Lamivudina/Nevirapina
E. Indinavir/Didanosina

CLAVE: E

29. Cual agente muy probablemente cause anemia y neutropenia aadidas


si se administran a un paciente con SIDA que toma Zidovudina:
A. Aciclovir
B. Amantadina
C. Ganciclovir
D. Pentamidina
E. Estavudina

CLAVE: C

30. Poco ms del 90% de este agente se excreta inalterable en la orina. Ya


que su solubilidad urinaria es baja, los pacientes deben ser bien
hidratados para evitar nefrotoxicidad:
A. Aciclovir
B. Amantadina
C. Indinavir
D. Zanamivir
E. Zidovudina

CLAVE: A

FIEBRE Y FIEBRE DE ORIGEN


DESCONOCIDO

FIEBRE DE ORIGEN DESCONOCIDO


DEFINICION DE PETERSDORF Y BEESON (1961)
1.

TEMPERATURA MAYOR O IGUAL A 38.3 C COMPROBADA EN


DIVERSAS OCASIONES.

2.

DURACION MAYOR DE 3 SEMANAS.

3.

AUSENCIA DE DIAGNOSTICO DESPUES DE UNA SEMANA DE


ESTUDIO HOSPITALARIO O UNA SEMANA DE ESTUDIO INTENSIVO E
INTELIGENTE EN CONSULTA EXTERNA.

125

FIEBRE DE ORIGEN DESCONOCIDO


(Durack-Street 1991 Curr Clin Top Inf Dis)

CLASIFICACION DE DURACK Y STREET

FOD. CLASICA
FOD. EN INFECCION POR HIV
FOD. EN PACIENTE NEUTROPENICO
FOD. NOSOCOMIAL

126

FOD CLASICA:

ENFERMEDADES INFECCIOSAS
ABSCESOS ABDOMINALES Y PELVICOS
INFECCION DE LA VIA BILIAR
TBC EXTRAPULMONAR
ENDOCARDITIS INFECCIOSA (Cr Duke)
PROSTATITIS
ESPONDILITIS Y OSTEOMIELITIS
SINUSITIS
ABSCESOS PERIDENTARIOS
INFECCION VIRAL SISTEMICA
PALUDISMO

127

FOD CLASICA: NEOPLASIAS


LINFOMAS
LEUCEMIAS
HIPERNEFROMA
CARCINOMA DE COLON
HEPATOMA
CA PULMON CELULAS PEQUEAS
CARCINOMA DE PANCREAS
SARCOMAS RETROPERITONELES
MIXOMA AURICULAR

128

FOD VASCULITIS Y COLAGENOSIS


ENFERMEDAD DE STILL
ARTERITIS DE CELULAS GIGANTES
VASCULITIS DE HIPERSENSIBILIDAD
PERIARTERITIS NODOSA
ENF MIXTA DEL TEJ. CONECTIVO
ARTERITIS TAKAYASU
ENF.DE BEHCET
GRANULOMATOSIS DE WEGENER

129

FOD:MISCELANEAS
HEPATITIS GRANULOMATOSA
SARCOIDOSIS
ENFERMEDAD DE CROHN
TROMBOFLEBITIS PELVICA
FIEBRE MEDICAMENTOSA
CIRROSIS HEPATICA
TIROIDITIS SUBAGUDA
HEMATOMA OCULTO
NEUMONITIS POR HIPERSENSIBILIDAD
HIPERTIROIDISMO
FIEBRE MEDITERRANEA FAMILIAR
FIEBRE FACTICIA

130

FOD EN PACIENTES HIV


INFECCION CONFIRMADA POR HIV.
TEMPERATURA MAYOR O IGUAL DE 38.3C COMPROBADA
EN DIVERSAS OCASIONES.
DURACION MAYOR DE 3 DIAS EN PACIENTE HOSPITALIZADO
DURACION DE 4 SEMANAS EN PACIENTE AMBULATORIO
AUSENCIA DE DIAGNOSTICO:
FRECUENTES:

POSTERIOR A 3 DIAS DE ESTUDIO ADECUADO


INCLUYENDO EN ELLOS 2 DIAS DE INCUBACION DE
CULTIVOS MICROBIOLOGICOS.

TUBERCULOSIS
CRIPTOCOCOSIS
MYCOBACTERIA ATIPICA

131

FOD NOSOCOMIAL
FIEBRE MAYOR DE 38C EN PACIENTE
HOSPITALIZADO. (24 HORAS)
AUSENCIA EN EL MOMENTO DEL INGRESO
DE INFECCION O INCUBACION
DE
ELLA.
AUSENCIA DE DIAGNOSTICO TRAS 3 DIAS
DE ESTUDIO ADECUADO
INCLUYENDO 2
DIAS DE CULTIVOS MICROBIOLOGICOS.

ENTIDADES MAS
FRECUENTES
CANDIDIASIS SISTEMICA
CITOMEGALOVIRUS
HEPATITIS ANICTERICA
COLECISTITIS ALITIASICA
SINUSITIS MAXILAR POR SNG
FIEBRE POR MEDICAMENTOS
COLITIS PSEUDOMEMBRANOSA
EMBOLISMO PULMONAR

132

FOD NEUTROPENICOS
RELACION ENTRE EL NUMERO DE
NEUTROFILOS (< 500 PMN) Y FRECUENCIA
DE INFECCIONES.
ENFERMEDAD DE FONDO Y DURACION DE
NEUTROFILIA
ENTIDADES:

BACTEREMIA
NEUMONIA GRAVE
INFECCION DE PIEL Y TEJIDOS
BLANDOS
INFECCIONES VIRALES SISTEMICAS
MICOSIS SISTEMICA

133

134

INDICACIONES DE HEMOCULTIVO

135

INDICACIONES DE HEMOCULTIVO

136

31. Seale cul de las siguientes afirmaciones es correcta en relacin con


el manejo de los hemocultivos en adultos:
A. Se deben practicar en todos los pacientes que acudan a urgencias con
fiebre >39C.
B. La extraccin a travs de catter venoso es ms fcil y rentable.
C. Se deben extraer, con un intervalo de 30-60 minutos, tres muestras de
10-30 ml.
D. Una vez extrados, se deben guardar en nevera hasta su procesamiento.
E. No son de utilidad en pacientes con enfermedades autoinmunes.

CLAVE: C

32. Paciente varn de 28 aos que presenta 9 das de fiebre y malestar general. No
refiere antecedente de hospitalizacin ni enfermedades crnicas previas. Examen
fsico T 40C, resto de examen fsico sin alteracin. Lab: leucocitosis, VSG,
Trombocitosis, hemocultivos (-), PPD (-), Ziehl-Neelsen de esputo (-), aglutinaciones
para S. typhi y Brucella spp. (-), resto de exmenes de laboratorio no mostraron
alteracin. Cul de las siguientes definiciones se ajusta MEJOR al presente caso?:
A. Fiebre de origen desconocido clsico
B. Fiebre prolongada sin foco aparente
C. Fiebre de origen desconocido en paciente neutropnicos
D. Fiebre de origen desconocido en paciente con VIH
E. Fiebre no prolongada sin foco aparente

CLAVE: B

33. Causa ms frecuente por clasificacin FOD a nivel infeccioso:


A. Brucelosis.
B. Sinusitis subaguda.
C. Tuberculosis.
D. Sndrome Retroviral Agudo.
E. Meningitis.

CLAVE: C

BACTEREMIAS Y SEPSIS

Definiciones
Infeccin: Presencia de microorganismos en un sitio normalmente estril, usualmente acompaado de
respuesta inflamatoria del husped.
Bacteriemia: Presencia de bacteria en sangre (confirmada por hemocultivo).
Sndrome de respuesta inflamatoria sistmica (SRIS): Respuesta sistmica a una amplia variedad de
condiciones clnicas, infecciosas (sepsis) y no infecciosas (quemaduras, pancreatitis) manifestada por 2
+ de las condiciones siguientes:
Temperatura >38C <36C;
Frec. Cardaca >90/min;
Frec. Respiratoria >20/min PaCO2 <32 mmHg;
Leucocitos >12,000xmm3 <4,000xmm3 abastonados >10%
Sepsis:
Evidencia clnica de infeccin, ms
Evidencia de SRIS
Sndrome Sepsis:
Sepsis, ms
Evidencia de perfusin alterada de rgano (al menos uno de los siguientes): hipoxemia, lactato
elevado, oliguria, alteracin estado mental.
142

Definiciones

Sepsis severa: Sepsis asociada a disfuncin de rgano,


hipotensin, anormalidades de perfusin.

Shock sptico: Sepsis con hipotensin a pesar de adecuado


reto de fludos en presencia de anormalidades de perfusin.
Shock sptico refractario: Shock sptico que no responde
durante 1 hora ms a administracin de fludos intervencin
farmacolgica.

143

SNDROME DE DISFUNCIN ORGNICA:

DISFUNCIN CEREBRAL:
Uno o ms de los siguientes criterios:
1. Agitacin psicomotora.
2. Desorientacin.
3. Somnolencia.
4. Convulsiones.

DISFUNCIN PULMONAR:
PaO2/FIO2 < 200
DISFUNCIN HEPTICA: dos o ms de los siguientes criterios:
1. Blirrubinas totales > 3
2. Transaminasas > 100
3. PT > 1.5 veces el control normal.

DISFUNCIN GASTROINTESTINAL:
Sangrado digestivo

144

145

34. Paciente que ingresa con fiebre de 39C, ictericia y sntomas urinarios.
Leucocitos 12.000 x mm3 con 10% de abastonados y plaquetopenia.
Presenta PA: 80/50 mmHg, pulso: 125 por minuto, que mejoran con
fluidoterapia. El diagnstico es:
A. Sepsis
B. Sepsis severa
C. Shock sptico
D. Infeccin urinaria baja
E. Sndrome de respuesta inflamatoria sistmica

CLAVE: B

35. Son criterios para definir Sndrome de Respuesta Inflamatoria


sistmica:
A. Temperatura >38 C < 36C, FC> 90 por minuto y FR >20 por
minuto.
B. Temperatura >38.5 C, FC > de 100 por minuto y FR >.20 por minuto.
C. Temperatura >38.5 FC> de 120 por minuto y FR > 24 por minuto.
D. Temperatura >38 C, FC> de 100 por minuto y FR > 24 por minuto.
E. Temperatura >38.5 C FC> de 100 por minuto y FR. > 32 por minuto

CLAVE: A

36. Indique el patrn hemodinmico clsico del shock sptico:


A. Hipotensin, GC alto, RVS baja
B. Hipotensin, GC alto, RVS normal
C. Hipotensin, GC normal, RVS elevada
D. Hipotensin, GC normal, RVS bajo
E. Hipotensin, GC bajo y RVS alta

CLAVE: A

INFECCIONES INTRAHOSPITALARIAS

Infecciones asociadas a la atencin en salud (IAAS)


INFECCIONES INTRA-HOSPITALARIAS

Definicin:

Infeccin que se adquiere luego de 48 horas de


permanecer en el Hospital y que el paciente no portaba a
su ingreso. Se consideran tambin a aquellos procesos
infecciosos que ocurren hasta 30 das luego del alta

150

INFECCIONES INTRAHOSPITALARIAS:

FORMAS TRANSMISION INFECCIONES


AIRE(<5u):
HANTA

TB, SARAMPION, VARICELA ZOSTER,

GOTAS(>5u): influenza , neumococo, otros


CONTACTO:

Directo: Persona - Persona.


Indirecto: MANOS, INSTRUM,OBJETOS.

VEHICULO COMUN (Medicinas, Alimento)


VECTORES
151

NEUMONIAS INTRAHOSPITALARIAS
50% de IIH en UCI
La letalidad general de la infeccin flucta entre 30 50%, y depende tambin de la bacteria
causante.
Es de alrededor de 50% para las neumonas causadas por gram(-), LAS MAS FRECUENTES y de
520% para las causadas por gram (+).
Para las neumonas causadas por P. aeruginosa, la tasa de letalidad puede alcanzar el 70%.
Factores de riesgo:

Edad avanzada
Tabaquismo
Obesidad
Antiacidos

Malnutricion
Alcoholismo
Alteracion de conciencia
Cirugia previa

Ventilacion asistida

152

37. La va ms habitual de transmisin de los microorganismos


relacionados con es infeccin nosocomial es:
A. Aire
B. Fmites.
C. Exposicin a una fuente comn.
D. Insercin de catteres.
E. Contacto directo con las manos.

CLAVE: E

38. Cul es la principal medida de prevencin para el control de


infecciones intrahospitalarias?
A. Lavado de manos
B. Vacuna contra hepatitis B
C. Uso de guantes
D. Uso de mascarillas
E. Uso de mandiles

CLAVE: A

39. El agente etiolgico ms frecuente en la neumona intrahospitalaria es:


A. Legionella pneumophylia.
B. Bacterias grampositivas.
C. Bacteria anaerbicas.
D. Micoplasma pneumoniae.
E. Bacterias gramnegativas.

CLAVE: E

ENDOCARDITIS INFECCIOSA

Endocarditis Infecciosa
Velo
valvular

La endocarditis infecciosa es una


infeccin microbiana que afecta a
la superficie endotelial del corazn

Cicatrizacin

Lesin
Endotelial

La lesin mas caracterstica es la

vegetacin constituida por una


masa amorfa de plaquetas y fibrina
de tamao variable que contiene
microorganismo mltiples y
escasas clulas inflamatorias.

Endocarditis
infecciosa
(vegetacin)

Agregacin
Plaquetaria

Colonizacin

Fibrina

ETA/ETNB

Bacteriemia

CLASIFICACION ACTUAL
EI segn la ubicacin de la infeccin y la presencia o
ausencia de material intracardiaco
1. EI de vlvula nativa izquierda
2. EI de vlvula protsica izquierda (EVP)
EVP precoz: < 1 ao tras la ciruga de la vlvula
EVP tarda: > 1 ao tras la ciruga de la vlvula

3.-EI derecha

ETIOLOGIA DE LA ENDOCARDITIS
INFECCIOSA

a)Endocarditis de
vlvula nativa:

Streptococcus spp. 45- 60%


Streptococcus viridans 30- 40%
Enterococcus spp 5- 18%
Staphylococcus aureus 10- 27%

b) Endocarditis

infecciosa
en adictos por va
endovenosa:

vlvulas derechas
tricspide.
Microbiologa:
Staphylococcus aureus 50%
Streptococcus spp 15%
Bacilos aerobios gram - 15%
Hongos 5%
Polimicrobianas 5%
Cultivo negativo 5%

ETIOLOGIA DE LA ENDOCARDITIS
INFECCIOSA
c) Endocarditis

protsica:

Temprana:
Estafilococos coagulasa
negativos 30%
Staphylococcus aureus 20%
Bacilos gram negativos 20%
Hongos 10%
Difterioides 5%
Enterococcus spp 5-10%
Streptococcus grupo viridans
<5%

Tarda:
Streptococcus grupo viridans
25%
Estafilococos coagulasa
negativos 20%
Staphylococcus aureus 10%
Bacilos gram negativos 10%
Hongos 5%

ENDOCARDITIS INFECCIOSA
CULTIVO NEGATIVO
Antibiticoterapia previa.
Variantes nutricionales
de Streptococcus.
Coxiella burnetti
Organismos HACEK.

Haemophylus aphrophilus
Actinobacillus actinomycetecomitans
Cardiobacterium hominis

Eikenella corrodens
Kingella kingae

ENDOCARDITIS INFECCIOSA
TRATAMIENTO EMPRICO

40. Cul es el agente etiolgico de la endocarditis infecciosa en usuarios


de drogas endovenosas?:
A. Klebsiella pneumoniae.
B. Enterococos faecalis.
C. Estreptococos viridans.
D. Neisseria gonorrhoeae.
E. Staphylococcus aureus.

CLAVE: E

41. La endocarditis protsica tarda se debe principalmente a:


A. Pseudomona cepacia.
B. Estreptococo viridans.
C. Estafilococo aureus.
D. Estafi lococo epidermis.
E. Cndida albicans.

CLAVE: B

42. En la endocarditis infecciosa Cul entre los que se mencionan, es el


germen responsable con ms frecuencia de los casos con hemocultivo
negativo?:
A. Estafi lococo meticilino-resistente
B. Enterococo
C. Estreptococo salivarius
D. Coxiella burnetti
E. Bacteroides fragilis

CLAVE: D

43. El microorganismo ms frecuente de endocarditis infecciosa en la


actualidad es:
A. Microorganismos del grupo HACEK
B. Estafilococo aureus
C. Estafilococo coagulasa negativos
D. Bacilos gramnegativos
E. Estreptococo viridans

CLAVE: B

INFECCIONES DEL APARATO RESPIRATORIO Y NEUMONIAS

FARINGITIS AGUDA
Sndrome Inflamatorio de Faringe.
En su mayoria producido por Virus.
Infeccion bacteriana mas importante: Estreptococo Bhemoltico.

Viral:
Rinovirus (resfrio comun) 20%
Coronavirus
6%
Adenovirus
5%
Parainfluenza
2%
Influenza
2%

Bacteriana:

Streptococo pyogenes (B-hemo. grupo A) 15-30%


Estreptococo B-hemo. grupo C
5-10%
Mixta por anaerobios
<1%
N. gonorrhoeae
<1%
Corynebacterium
<1%
Chlamydeas y Mycoplasmas
Desco.

DIFTERIA
Corynebacterium difteriae

Factor de virulencia
La toxina difteria es una exotoxina, una protena de 58.330
Kda de peso molecular
Induce la muerte de la clula blanco en el epitelio respiratorio
al detener la sntesis de protena.
El C. difteriae se localiza en la nariz o en la garganta para dar
inicio a la enfermedad.
La produccin de toxinas depende de se infeccin por un
bacterifago (lisogenizacion) y de un rango critico en la
concentracin de hierro en el medio.

Diagnostico
Es clnico observacin de pseudomembrana
Cultivo en medio de agar telurito, leffler y agar sangre.
inmunoflurescencia.

TRATAMIENTO
Antitoxina difterica
Erradicar la bacteria
Eritromicina 40 mg/kd dia durante 14 das
Penicilina G 300 000 U durante 14 das

TOS FERINA
Infeccin de la vas
respiratorias
.
Enfermedad producida por bacilo gram (-)

B. Pertusis
Tosferina

B. Parapertusis
Produce
cuadro
muy
parecido al coqueluche

TRATAMIENTO
Eritromicina a dosis de 30-50
mg/Kg/da
T. Sulfa a 10 mg/kg/da durante 10
das

NEUMONIA
EXTRAHOSPITALARIA
Neumona adquirida en la comunidad extrahospitalaria.
Neumona intrahospitalaria.
N. por aspiracin
S pneumoniae, S piogenes, M pneumoniae, H influenzae, M catharralis
P gingivalis, P melaninogenicus, F nucleatum, Actinomicetos, espiroquetas
Gram negativos

N. bacterianas no neumoccicas
M pneumoniae, C burnetti, C psittachi, K pneumoniae, L pneumoniae

N. vricas (neumonitis)
Influenza, varicela-zoster, citomegalovirus

N. eosinfilas
Sd de Loeffler

ETIOLOGA MS FRECUENTE
Tipo de paciente
Extrahospitalario

Etiologa
S. pneumoniae
Micoplasma pneumoniae
Haemophilus influenzae
Chlamydophila pneumoniae
Virus respiratorios*

Ingresado(No UCI)

S. pneumoniae
M. pneumoniae
C. pneumoniae
H. influenzae
Legionellas
Aspiracin
Virus respiratorios*

Paciente en UCI

* Influenza A y B, Adenovirus,
Sincitial respiratorio y
Parainflueza

S. pneumoniae
S. aureus
Legionellas
Bacilos gramnegativos
IDSA/ ATS Guidelines for CPA in Adults. CID2007:44(Suppl 2). S

CRITERIOS DE INGRESO HOSPITALARIO.


NEUMONA DE LA COMUNIDAD
1. Score de severidad

Confusin

Uremia>7 mmos/l

CURB-65

FR> 30 X min

Edad>65 aos

TAS< 90, D< 60mmhg

CURB 0-1 TTO extrahospitalario


CURB 2 _ Ingreso hospitalario
CURB>=3 Ingreso en UCI

Ms reciente modificacin de los criterios de la BTS, de un anlisis multivariado de 1068 ptes, que identificaron
estos 5 factores indicadores de mortalidad: Por ejemplo: Si CURB 0- O.7%, Si 3, 4, , 5- 14.5%, 40% y 57%
respectivamente

IDSA/ ATS Guidelines for CPA in Adults. CID2007:44(Suppl 2). S 27

NEUMONIA
FACTOR PREDISPONENTE Y/O ANTECEDENTES DE IMPORTANCIA:
-TABAQUISMO
:
H. influenzae
-ASPIRACIN

-EPIDEMIAS o
RECLUTAMIENTO

ANAEROBIO

VIRAL O MICOPLASMA

-MAYORES DE 65 AOS :

GRMENES MS AGRESIVOS

- ALCOHOLISMO

K. pneumoniae

RECOMENDACIN EMPRICA - EEUU


Pacientes extrahospitalarios

Pacientes en UCI

Macrlidos( Nivel I de evidencia)


Doxiciclina(Nivel II de evidencia)
Comorbilidades como cardiopata, neumopata,
enfermedad heptica o renal, alcoholismo, diabetes
mellitus, neoplasias, asplenia, inmunosupresin
FQ respiratoria (Moxifloxacino, Gemifloxacino, o
Levofloxacino)Nivel I de evidencia
Un Betalactmico + Macrlido ( Nivel I de
evidencia)

Un Betalactmico ( Cefotaxime, Ceftriaxona, o


Ampicilln/Sulbactam) + Azitromicina ( Nivel II de evidencia)
o una FQ respiratoria( Nivel I de evidencia).
Consideraciones especiales
Si sospecha de Pseudomona: Un Betalactmico
antineumoccico y antipseudomnico
(Piperacilina+Tazobactam, Cefepime, Imipenem, o
Meronem) + Cipro o Levofloxacino

Pacientes ingresados fuera de UCI


FQ respiratoria ( Nivel I de evidencia)

Un Betalactmico + Macrlido ( Nivel I de evidencia)

Si sospecha de Neumona Comunitaria por SAMR: Adicionar


Vancomicina o Linezolid

IDSA/ ATS Guidelines for CPA in Adults. CID2007:44(Suppl 2). S 27

SPEIT

44. Con respecto a las infecciones vricas respiratorias:


A. Se complican con infecciones bacterianas secundarias.
B. La mayora son autolimitadas y banales.
C. Requieren un diagnostico microbiolgico.
D. Estn producidas por un nmero reducido de virus.
E. Responden bien al tratamiento con un antiviral especfico

CLAVE: B

45. Paciente mujer de 60 aos sin antecedente de importancia que acude a la


emergencia presentando 3 das de fiebre, odinofagia, edema de cara lateral
izquierda de cuello y adenopata cervical izquierda. Al examen se aprecia placas
blancas grisceas en la regin amigdaliana izquierda que luego de ser raspada con
baja lengua se aprecia lecho sangrante. Los exmenes de laboratorio mostraron
leucocitosis con neutrofilia y desviacin izquierda y un gram que mostr bacilos
gram positivos, resto de exmenes sin alteracin. Cul es el agente etiolgico ms
probable?:
A. S. pneumoniae.
B. Parainfluenza.
C. Corinebacterium difteriae.
D. Bordetella pertussis.
E. Moraxella catarrhalis.

CLAVE: C

46. Paciente varn de 54 aos con antecedentes de DM tipo 2 y alcoholismo.


Presenta fiebre, tos y expectoracin sanguinolenta y purulenta de 2 das. Rx
trax: Infiltrado neumnico en lbulo superior derecho con abombamiento de
la cisura interlobar superior y leve derrame pleural. Lab: Leucocitosis con
desviacin a la izquierda. La etiologa probable del proceso neumnico
CORRESPONDERIA a:
A. Estreptococo pneumoniae.
B. Estafi lococo aureus.
C. Pseudomona aeuriginosa.
D. Haemophilus infl uenzae.
E. Klebsiella pneumoniae.

CLAVE: E

47. De los siguientes agentes etiolgicos, en cul de ellos es


caracterstico la disociacin clnica radiolgica?
A. Staphylococcus aureus.
B. Haemophilus infl uenzae.
C. Klebsiella pneumoniae.
D. Moraxella catarrhalis.
E. Mycoplasma pneumoniae

CLAVE: E

48. Un paciente con bronquitis crnica ingresa a hospitalizacin con


neumona bilateral extensa e insuficiencia respiratoria severa. Qu
pauta antibitica sera la ms adecuada?:
A. Penicilina y Eritromicina
B. Ceftriaxona
C. Ceftazidima y Amikacina
D. Penicilina ms Gentamicina
E. Imipenem

CLAVE: C

TUBERCULOSIS

TB PULMONAR (70 93%): Tos productiva, raramente hemoptisis, dolor


torcico y dsnea.
En los severamente INMUNOCOMPROMETIDOS:
Compromiso pulmonar basal
Pneumona tuberculosa
Linfa adenopata hiliares o mediastinales
TB miliar.
TB larngea es la mas bacilifera
Tuberculosis digestiva, mas frecuente en regin ileocecal, luego el
ciego.
Tuberculosis osteoarticular: columna dorsal

POBLACIONES BACILARES
BACILOS DE MULTIPLICACION
RAPIDA:
medio ptimo el extracelular
(pH 6.5).
Maxima oxigenacin: paredes
de cavernas.
Gran cantidad de bacilos por lo
que tiene Gran posibilidad de
mutantes resistentes.
H
BACILOS DE MULTIPLICACION
INTERMITENTE:
Condiciones desfavorables.
Caseum slido. Extracelulares.
Poblacin < 105
Capacidad de recidiva
R

BACILOS DE MULTIPLICACION
LENTA: localizacin
intramacrofgica. Ph acido.
Poblacin : < 105

BACILOS EN ESTADO
LATENTE
No susceptibles a
frmacos
Reactivaciones y
recidivas

Poblacin
Cavitaria

INH

POBLACIONES BACILIFERAS Y ACTIVIDAD DE DROGAS


ANTITUBERCULOSIS
Poblacin
Intracelular

RFP
PZA

SM

Poblacin
Intracseum

RFP
(EMB)

INH
EMB

Crecimiento
Geomtrico

Crecimiento
Lento

RFP

Poblacin
en reposo

INH
?

Crecimiento
Intermitente

Crecimiento
Anaerbio?

TUBERCULOSIS : TERAPIA

AGENTES ESPECIFICOS
ISONIACIDA

BACTERICIDA(SNC) 5 mg/kg

HEPATITIS (TGO,TGP)
NEUROAPATIAS.

RIFAMPICINA

BACTERICIDA(SNC) 10 mg/kg

COLORACION DE FLUIDOS
HEPATOTOXICIDAD
HIPERSENSIBILIDAD

ESTREPTOMIC. BACTERICIDA
ETAMBUTOL

BACTERIOSTATIC

PIRAZINAMIDA BACTERICIDA(IC)

15 mg/kg

HIPERSENSIBILIDAD
OTOTOXICIDAD. FALLA RENAL

20 mg/kg
NEURITIS OPTICA
HIPERSENSIBILIDAD
25 mg/kg
HEPATOTOXICIDAD
HIPERURICEMIA

Corticoides en el
tratamiento de la
tuberculosis:

1. Tuberculosis
menngea
2. Tuberculosis
pericrdica

ESQUEMA ACTUAL

TBC MDR
La TBC MDR se define como la tuberculosis causada por cepas de
Mycobacterium tuberculosis, que son resistentes a :
Ncleo del tratamiento de la TB sensible: R-H

TBC XDR
Tuberculosis extremadamente resistente (TB XDR) se define como la resistencia
simultnea a los dos ncleos principales para el tratamiento de la TB:
Ncleo del tratamiento de la TB sensible: R-H
Ncleo del tratamiento de la TB MDR: quinolona + inyectable de 2 lnea
(Kanamicina, Capreomicina o Amikacina)

49. Sobre el tratamiento antituberculoso, correlacione:


1) Isoniazida,
2) Rifampicina,
3) Pirazinamida,
4) Estreptomicina,
5) Etambutol;
( ) Dosis de 5 mg/kg/dia,
( ) Disfuncin vestibular y coclear,
( ) Reduccin de la agudeza visual,
( ) Dosis 10mg/kg/dia,
( ) Dosis 25 mg/k/dia
A. 1,2,4,5,3
B. 1,2,3,5,4
C. 1,4,2,3,5
D. 1,5,4,2,3
E. 1,4,5,2,3

CLAVE: E

50. En el tratamiento antiTBC el frmaco que acta como bacteriosttico en


bacilos en reposo y como bactericida en fase de multiplicacin rpida
es:
A. Etionamida
B. Estreptomicina
C. Etambutol
D. Pirazinamida
E. Isoniazida

CLAVE: E

51. Mujer de 17 aos con diagnstico de LES grave y tratamiento con


esteroides a dosis elevadas. Radiografa de trax: normal. PPD:
12x15mm de induracin. La actitud a seguir es:
A. Efectuar profilaxis con Isoniazida 6 meses
B. Esperar 2 aos para realizar quimioprofilaxis
C. No utilizar quimioprofilaxis
D. Trata con Rifampicina ms Isoniazida
E. Tratar con Rifampicina, Isoniazida y Etambutol.

CLAVE: A

52. La tuberculosis asociada a la infeccin por VIH se caracteriza por:


A. Presentacin subclnica de la enfermedad
B. Aparicin caracterstica en los estadios de inmunodepresin ms severa
C. Elevada frecuencia de afectacin extrapulmonar y diseminada
D. Escaso rendimiento de los mtodos microbiolgicos de diagnstico
E. Mala respuesta al tratamiento antituberculoso

CLAVE: D

INFECCIONES DEL TRACTO DIGESTIVO

CAMPYLOBACTERIOSIS
Bacteria Gram negativa mvil, de forma espiralada.
Infeccin comn en el primer ao de vida.
Campylobacter jejuni (80%)
Campylobacter coli (15%)
Signos y sntomas
Inflamacin
Dolor abdominal
Diarrea sanguinolenta con moco
- Guillian Barre (asociado)
Productos avcolas (pollos y otras aves)
Contaminacin cruzada de alimentos con productos
avcolas contaminados.
Los macrolidos (azitromicina) y la ciprofloxacina son los
antibiticos de primera y segunda eleccin.

VIBRIO CHOLERAE
Epidemias en pases en vas de desarrollo.
Diarrea voluminosa deshidratante
Antibiticos acortan el tiempo de enfermedad
Toxina
colrica:
compuesta
por
cinco
subunidades B (de enlace) y una subunidad A
(activa). Las subunidades B se unen a los
receptores del gangliosido GM1.
se separan la subunidad A1 y el componente A2
El componente A1 de la toxina colerica estimula la
produccion
de
la
enzima
adenilciclasa,
involucrada en la produccion del monofosfato
ciclico de adenosina (AMPc).

Tratamiento: Doxiciclina

adenilatociclas
a

Diarrea del viajero

198

PERIODO DE INCUBACION y FUENTE DE INFECCION

FIEBRE TIFOIDEA

200

Tratamiento

Laboratorio
Biometra hemtica
Reaccin de Widal
Hemocultivo 80 %
Mielocultivo 90%
Coprocultivo,urocultivo

201

53. En las 4 horas siguientes a una cena parroquial, 25 personas


manifestaron la brusca aparicin de nauseas, vmitos y clicos
abdominales, el agente infeccioso MS probable ser:
A. Enterotoxina estafiloccica.
B. Toxina de Clostridium botulinum.
C. Escherechia coli enterotxica.
D. Clostridium perfringens.
E. Salmonella typhimurium.

CLAVE: A

54. Mujer de 19 aos que se fue de viaje a la selva con sus compaeras de la
residencia, regres hace 3 semanas y presenta dolor abdominal en
hipogastrio y diarrea, el dolor aumenta en intensidad con el tiempo como
tambin las deposiciones (10 al da con moco y sangre). Al examen: Afrebril,
abdomen doloroso a palpacin en FII. El microorganismo causal MS
probable:
A. E. coli
B. Salmonella
C. Shigella
D. V. parahaemolyticus
E. E. histolytica

CLAVE: E

55. En un paciente con fiebre tifoidea lo CORRECTO es:


A. La complicacin ms severa es enterorragia
B. El antimicrobiano de primera eleccin es Cotrimoxazol.
C. El bilicultivo es el mtodo de mayor sensibilidad para el diagnstico.
D. La perforacin intestinal slo requiere tratamiento mdico.
E. La infeccin crnica es infrecuente

CLAVE: A

56. Entre los mtodos de diagnstico etiolgico de la fiebre tifoidea, el que


tiene mayor positividad es:
A. Mielocultivo.
B. Hemocultivo.
C. Cultivo de bilis.
D. Urocultivo.
E. Ninguno de los anteriores.

CLAVE: A

INFECCIONES DE LAS PARTES BLANDAS

FOLICULITIS
(S. aureus)
Pstulas foliculares favorecidas
por maceracin y oclusin en
cara, pelo y extremidades.

FORNCULO
(S. aureus)
Ndulo inflamatorio doloroso a partir
de foliculitis con celulitis perilesional

IMPTIGO

ECTIMA

Infeccin superficial

Lesin ms profunda de Imptigo (o Imptigo


ulcerado).

Staphylococcus aureus o
Estreptococos -hemolticos grupo A.

Etiologa
Estreptococo B- hemoltico Grupo A
Estafilococo aureus

Manifestaciones clnicas
Ulceraciones redondeadas, aspecto en
"sacabocado" cubiertas por costra amarillenta.
Localizacin usual en las piernas.

ERISIPELA
Infeccin aguda de la piel y los tejidos
subcutneos.
Etiologa: Estreptococo pyogenes
Manifestaciones clnicas
Inicio sbito de fiebre alta, cefalea,
malestar y vmitos.
Piel en sitio afectado roja, sensible y
edematosa ("piel de naranja")
El eritema est muy bien definido,
bordes elevados

CELULITIS
Inflamacin del tejido conectivo laxo,
en particular del subcutneo.
Etiologa: Estreptococo B hemoltico
grupo A, Estafilococo aureus
Clnica: Eritema sensible con bordes mal
definidos no elevados. Puede surgir en
piel normal o iniciar por una grieta
superficial.

GANGRENA GASEOSA
Es una mionecrosis asociado a toxicidad sistmica
C. perfringes: en el 80% de casos
C. septicum (asociado a Ca. colon)
A-Toxina: la ms importante es una fosfolipasa, hemlisis
masiva, dao capilar y destruccin de plaquetas

Manejo:
Lo mas importante: debridamiento extenso
Se tiene que debridar todo el msculo
necrtico.
requiere cirugas diarias
Tratamiento antibiotico:
Penicilina G sdica
Si alergia: Imipenem, Metronidazol, Cloramfenicol
Antitoxina para Gangrena Gaseosa
Oxigeno Hiperbrico

FASCEITIS NECROTIZANTE

Penicilina G +
Cefalosporina Gram (-) +
Metronidazol o Clindamicina

FASCEITIS NECROTIZANTE y GANGRENA GASEOSA

57. Paciente con quemaduras de tercer grado, presenta fiebre persistente y


compromiso del estado general. De las heridas se aisl Pseudomona
aeuriginosa. Cul de los siguientes antibiticos es el ms apropiado?:
A. Cefaclor.
B. Cefalotina.
C. Cefoxitima.
D. Ceftazidima.
E. Cefuroxima.

CLAVE: D

58. Una mujer de 90 aos con demencia, incontinente e incapacitada por


hemiparesia, presenta una lcera sacra de grado III. En la exploracin vemos
que est en la cama sobre una almohadilla. Est afebril y tiene un pulso y
una presin arterial normal. Se evidencia una lcera sacra de 4 x 4 cm que
se extiende hacia la fascia con exudado verde y piel normal que rodea a la
lcera. Cul es la primera prioridad en los cuidados generales de esta
paciente?:
A. Empezar tratamiento con antibiticos.
B. Cultivar el exudado de la lcera
C. Aplicar vendajes hmedos de solucin salina tres veces al da.
D. Hacer cambios posturales a la paciente cada dos horas.
E. Colocar una sonda urinaria permanente

CLAVE: D

59. Paciente varn de 54 aos con antecedente de diabetes desde hace 10 aos
que toma medicacin irregularmente. Acude a la emergencia del HNHU
presentando 1 da de dolor y flogosis en regin escrotal y perianal. Al
examen fsico se encontr dolor y empastamiento de partes blandas regin
escrotal y perianal, asimismo impresiona crpitos durante la palpacin de
dicha zona que se extiende hacia glteos. Qu rgimen de antibiticos
elegira?
A. Penicilina G + clindamicina.
B. Penicilina benzatinica + clindamicina.
C. Meropenem + Ceftazidima.
D. Penicilina G + clindamicina + Ceftazidima.
E. Penicilina benzatinica + Ceftazidima.

CLAVE: D

INFECCIONES POR MORDEDURAS Y ARAAZOS

FIEBRE POR ARAAZO DE GATO


Bartonella henselae
Bacilo gram-negativo aerobico

araazo a lesion primaria 7 a 12 dias


Lesion primaria a linfadenopatia (5 a 50) dias media 12dias

Presentacin: Ganglios Regionales:


Axila
Cabeza y cuello
Inguinal

Entre 25% y 60% de pacientes reportan una lesin de


inoculacin primaria en piel (ppula o pstula de 0.5
1 cm).

En casos severos/complicados, se
puede prescribir TMP/SMX,
ciprofloxacina o azitromicina.

Mordedura de Perro

Se infectan del 15 al 20%.


Los grmenes ms frecuentes son:

Capnocytophaga canimorsus
Estreptococo hemoltico alfa
Pasteurella canis
Estafilococo aureus

Mordeduras de Gatos

Se infectan hasta el 80% de las heridas. El germen ms habitual es la


Pasteurella multocida.

Mordedura de perros, gatos y humana

se debe administrar antibiticos en forma profilctica:


Amoxicilina/clavulnico 875/125 VO cada 12 hs. 50
(En caso de alergia a penicilina, dar clindamicina300 mg por da + fluoroquinolona.)
El tratamiento profilctico se debe realizar durante 3 a 5 das.

60. Mientras jugaba, una nia de 8 aos es mordida por un gato negro.
Llega al mdico al da siguiente da con fiebre y dolor seo localizado
en su pantorrilla derecha. Los resultados del cultivo seo estn
pendientes. El patgeno ms probable es:
A. Brucella melitensis
B. Eikenella corrodens
C. Francisella tularensis
D. Pasteurella multocida
E. Yersinia pestis

CLAVE: D

61. Nio presenta mordida de su pitbull en pierna derecha. La herida es


limpiada y recibe 1 inyeccin de toxoide tetnico y 1 inyeccin de
Penicilina G. Das despus, herida se encuentra y purulenta. El
exudado se cultiva en agar de sangre y arroja bastones gram (-). Las
pruebas de sensibilidad ATB estn pendientes. Probable patgeno:
A. Bartonella henselae
B. Brucella canis
C. Clostridium tetani
D. Pasteurella multocida
E. Toxocara canis

CLAVE: D

INFECCIONES DEL SISTEMA NERVIOSO

AGENTES ETIOLGICOS SEGN EDAD Y


CONDICIONES DEL HUSPED para MENINGITIS
0-2 meses de edad:
Bacilos Gram negativos.
Estreptococos grupo B.
Listeria monocytogenes.
2 meses-6 aos de edad:
Neumococo.
Haemophilus influenzae.
Neisseria meningitidis.
> 6 aos de edad:
Neumococo.
Meningococo.

AGENTES ETIOLGICOS SEGN EDAD Y


CONDICIONES DEL HUSPED para MENINGITIS
M.B. post-neurociruga:
Bacilos gram negativos (80 %)
Estafilococo aureus y Estafilococos epidermidis (20%).
M.B. asociada a "Shunts" :
Estafilococo epidermidis (60 %)
Estafilococo aureus (30 %)
Bacilos gram negativos u otros (10 %).
M.B. recurrente:
Neumococo (80 %).

AGENTES ETIOLGICOS SEGN EDAD Y


CONDICIONES DEL HUSPED para MENINGITIS
Traumatismo del
SNC, ciruga

Staphylococcus aureus,
bacilos gram negativos.

Husped inmuno- Cryptococcus neoformans


comprometido
Candida albicans,
L. monocytogenes.

DIAGNOSTICO: diferenciacin bact / viral

TRATAMIENTO: Antibiticos.
- Emergencia infectolgica.
- Iniciar emprico inmediato a PL: > 3 hs de llegada al Hospital,

aumento significativo de mortalidad (Auburtin. CCM 2006; 34)

-Tratamiento emprico:
- Recomendacin:

Profilaxia de contactos:
Rifampicina 600 mg/d 2 dias
o Ciprofloxacino 500 mg/d una
dosis

Adulto < (50 ) - 60 aos: Ceftriaxona 2 g iv c/ 12 hs

Adulto > 50-60 aos: Ceftriaxona + Ampicilina 2 g c/4h incidencia de


Neumococo resist.: agregar Vancomicina

INFECCIONES VIRALES DEL SISTEMA NERVIOSO


Vas de entrada:
Nasal respiratoria:Parotiditis, sarampin, varicela, herpes simple 1

Cavidad bucal digestiva (intestinal): Enterovirus, Epstein barr


Mucosa genital: Herpes Simple 2
Inoculacin: Arbovirus, HIV

Transplacentaria: Rubola, Citomegalovirus HIV


Nervios perifricos: Herpes varicela-zoster, rabia
Una vez que el virus invadi el SN se multiplica en regiones selectivas del encfalo o
la mdula espinal, plexos coroideos y las meninges.

RABIA
El virus de la rabia pertenece al
gnero Lyssavirus de la familia
rabdoviridae.

Rabia urbana: el perro es el


reservorio principal en los pases
en desarrollo.

Rabia silvestre: el reservorio


principal de la rabia silvestre es el
murcilago hematfago
Desmodus rotundus

VIRUS DE LA RABIA
PATOGENIA
F
A
S
E
P
R
O
D
R

M
I
C
A

Inhalacin,
transplantes,
inoculacin

Mordedura
Msculo estriado

Nervios perifricos/SNC
Cerebro

FASE
NEUROLGICA

Piel de cuello y cabeza, glndulas salivales,


retina, crnea..

RABIA
Rabia furiosa
Sntomas inespecficos: agitacin, temor y
anorexia casi siempre con fiebre, agitacin e
irritabilidad.
Evoluciona: hidrofobia: espasmos inducidos
por la deglucin o visin del agua y la
aerofobia espasmos inducidos al soplar en la
cara del paciente.

Rabia paraltica
sntomas pueden ser parecidos a los del
sndrome de Guillain Barre.
fiebre persistente, la disfuncin sensorial
limitada a la regin de la mordedura o el
mioedema de perfusin (espasmo de un par
de segundos en una parte del msculo,
despertado con el martillo de percusin).

62.
Con respecto a los hallazgos en LCR correlacione:
a. MEC TBC,
b. MEC viral,
c. MEC bacteriana,
d. Todos,
e. a y c,
f. a y b;
( ) LCR Amarillo citrino,
( ) Tiene hipoglucorraquia,
( ) Hay aumento de celularidad,
( ) Predominio de PMN,
( ) Protenas normales-altas
A. A,c,d,e,f
B. A,e,d,C,b
C. A,b,c,f,b
D. B,c,d,f,b
E. B,e,c,f,b

CLAVE: B

63. La profilaxis de los contactos adultos de pacientes con meningitis


meningoccica , segn el MINSA es:
A. Tetraciclina 500 mg VO dosis nico
B. Rifampicina 600 mg VO c/12 horas por 2 das.
C. Ampicilina 500 mg VO c/6 horas por 3 das.
D. Sulfametoxazol/Trimetoprim 800/160 mg VO c/12 horas por 3 das.
E. Amoxicilina 500 mg VO c/6 horas por 5 das.

CLAVE: B

64. La manifestacin clnica ms frecuenta del absceso cerebral es:


A. Parlisis ocular
B. Fiebre
C. Letargo
D. Hemiparesia
E. Cefalea

CLAVE: E

65. El tratamiento de eleccin para un absceso cerebral secundario a


diseminacin hematgena por endocarditis infecciosa:
A. Gentamicina ms Cloramfenicol
B. Metronidazol ms Clindamicina
C. Cefotaxima ms Penicilina
D. Cefotaxima ms Metronidazol mas vancomicina
E. Cefotaxima ms Oxacilina

CLAVE: D

66. Cul de los siguientes enunciados es cierto en relacin con el cuadro


clnico de la rabia?:
A. El periodo de incubacin es largo, generalmente superior a un ao.
B. El periodo paraltico se denomina fase melanclica.
C. El periodo de excitacin se caracteriza por una parlisis flccida.
D. La rabia puede presentarse coma una paradoja ascendente similar al
sndrome de Guillan - Barr.
E. La hidrofobia es caracterstica del periodo prodrmico.

CLAVE: D

ENFERMEDADES DE TRANSMISION SEXUAL

Treponema pallidum
Espiroqueta 5-15 micras x 0,2 micras
Anaerobio estricto

Sfilis primaria

3 semanas despus de la infeccin.


lcera en el lugar de la infeccin llamada
chancro, indolora.
ganglios linfticos inflamados.
Normalmente, esta etapa dura de 2 a 6 semanas
y la lcera se cura.

SIFILIS
Sfilis secundaria

entre 6 semanas y 6 meses despus de la


infeccin.
erupcin cutnea que no pica, palmas de las
manos
Normalmente, esta etapa tiene una duracin
de unas pocas semanas

Sfilis latente

Sigue a la resolucin de la etapa


secundaria.
Las bacterias persisten y son
controladas, pero no son eliminadas
por el sistema inmunitario del husped.

Sfilis terciaria

Aparece en el 30% de los infectados, por lo general


dcadas despus de la infeccin.
Puede afectar cualquier parte del cuerpo.

242

SFILIS : DIAGNOSTICO POR LABORATORIO


DIRECTA: Treponema en campo oscuro
SEROLGICAS
NO TREPONMICAS

VDRL

( Venereal Disease Research Laboratory )

RPR

( Rapid Plasma Reagin )

TREPONMICAS

FTA-ABS ( Abs.Ac. treponmico fluorescente )


TPHA

( Trep. pall. hemaglutination )

ELISA
RIT

(Rabbit infectivity testing)

PCR

(Polymerasa chain reaction)

TRATAMIENTO

Estado

Tratamiento de eleccin

Tratamiento alternativo

Sfilis primaria, secundaria


y latente precoz

Penicilina benzatnica
2400,000 UI /IM, dosis
nica

Doxiciclina
Tetraciclina
Azitromicina

Sfilis latente tarda o de


duracin desconocida

Penicilina benzatnica
2400,000 UI /IM, semanal
por 3 semanas

Doxiciclina
Tetraciclina

Sifilis gestacional
Neurosfilis

En gestante : Penicilina
Benzatinica
Penicilina G acuosa, 1824 U/da por 10 14 das

Penicilina G acuosa

SD. FLUJO VAGINAL


Vaginitis : (Gardnerella vaginalis, Candida Albicans)
Metronidazol 2 gr. VO +
Clotrimazol 500 mg (OV)
Cervicitis: (Neisseria gonorrhoeae, Chlamydia trachomatis)
Ciprofloxacino 500 mg VO +
Azitromicina 1 gr. VO

245

SD. DESCARGA URETRAL


Agentes :
Neisseria gonorrhoeae: Perodo de Incubacin: 02 a 05 das,
Tincin con Gram (-) Se observan leucocitos PMN con
diplococos intracelulares Gram(-), en varones puede ocasionar
epididimitis
Chlamydia trachomatis Perodo de Incubacin: 01 a 03 sem.

Tratamiento
Ciprofloxacino 500 mg. VO +
Azitromicina 1 gr. VO Doxiciclina 100 mg. bid x 7 d.
246

SD. ULCERA GENITAL


Agentes : Treponema pallidum : Perodo de Incubacin: 2 a 6 semanas (a veces hasta 3 meses)
Haemophilus ducreyi: Periodo de incubacin 2 a 10 das. ulceras dolorosas a veces
mltiples, linfadenopatia regional habitualmente inguinal.
VHS 1-2
Tratamiento
Penicilina benzatnica 2,4 millones UI/IM +
Azitromicina 1 gr VO + Aciclovir 400 mg tid x 7d.

247

SD. DOLOR ABDOMINAL BAJO


Agentes : (Neisseria gonorrhoeae, Chlamydia trachomatis, grmenes
anaerobios)
Tratamiento
Ciprofloxacino 500 mg. VO
+
Doxiciclina 100 mg. VO bid x 14 d.
Metronidazol 500 mg. bid x 14 d.

Sndrome de Fitz-Hugh-Curtis es una perihepatitis producida por una peritonitis


secundaria al ascenso de bacterias, como resultado de una enfermedad inflamatoria
plvica
248

SD. BUBN INGUINAL


Agente : Chlamydia trachomatis: Perodo de Incubacin: 3 - 40 das
Tratamiento
Doxiciclina 100 mg. VO bid x 21 d

Eritromicina 500 mg. tid x 21 d.

249

SINDROME

TRATAMIENTO

Flujo Vaginal (Vaginitis)

Metronidazol 2 gramos VO stat +/Clotrimazol 500mg Ovulo Stat

Flujo Vaginal (Cervicitis)

Ciprofloxacino 500 mg VO
Azitromicina 1 gr. VO

Descarga uretral

Ciprofloxacino 500mg VO stat +


Azitromicina 1g VO stat

lcera genital

Penicilina G Benzatnica 2.4 millones UI IM +


Ciprofloxacino 500mg VO Stat

Aciclovir 400 mg tid x 7d.


Dolor abdominal bajo

Ciprofloxacino 500 mg. VO


+
Doxiciclina 100 mg. VO bid x 14 d. +
Metronidazol 500 mg. bid x 14 d.

Bubn inguinal

Doxiciclina 100mg VO c/12h por 14 das


250

67. Paciente de 35 aos que consulta por presentar disuria y secrecin uretral,
que conducta toma si el gram de la secrecin es negativo con abundantes
polimorfonucleares?:
A. Interpreta el cuadro como uretritis por chlamydia e indica azitromicina 1 g
dosis nica
B. Interpreta el cuadro como uretritis por gonococo e indica penicilina.
C. Interpreta el cuadro como uretritis por chlamydia e indica azitromicina 1 g/da
por cinco das.
D. Interpreta el cuadro como uretritis por gonococo e indica ceftriaxona 1 g dosis
nica.
E. Ninguna de las anteriores

CLAVE: A

68. Paciente de 23 aos que trabaja como DJ en un bar nocturno y tiene una
historia de promiscuidad bisexual e ingesta de drogas. Consulta por fiebre,
malestar general y dolor en el glande. La exploracin fsica muestra mltiples
tatuajes y piercings. A nivel genital presenta lesiones vesiculosas agrupadas
en glande y adenopatas inguinales bilaterales. El diagnstico MS probable
es:
A. Herpes genital.
B. Sfilis primaria.
C. Primo infeccin VIH.
D. Chancro blando.
E. Linfogranuloma venreo

CLAVE: A

69. Paciente varn de 54 aos que presenta desde hace 1 semana de


secrecin uretral dolorosa asociada a adenopata inguinal flogtica
dolorosas. Cul de los siguientes grmenes es posible agente
etiolgico?:
A. Treponema pallidum.
B. Clamidia trachomatis.
C. Neisseria gonorrhoeae.
D. Infeccin por VIH.
E. Cndida albicans.

CLAVE: B

70. Cul es el tratamiento para una epididimitis aguda en un varn de 60


aos, sexualmente activo?:
A. Eritromicina
B. Doxiciclina
C. Amoxicilina-clavulamico
D. Ofloxacino
E. Dicloxacilina

CLAVE: D

71. Paciente varn tratado con cefalosporina de 3ra generacin por


secrecin uretral. La secrecin contina y el cultivo sali negativo. El
agente etiolgico MS probable es:
A. Herpes tipo 1
B. C. trachomatis
C. C. psittaci
D. Gonococo resistente
E. Tricomona

CLAVE: B

72. Paciente mujer con VDRL (-) y FTA-ABS (+), el diagnstico MS


probable es:
A. Sfilis reciente no tratada
B. Sfilis reciente tratada
C. Sfilis antigua tratada
D. Falso positivo
E. A y C

CLAVE: E

INFECCIONES Y PROFESIONES

ENFERMEDAD DE LYME
Agente etiolgico: Borrelia burgdorferi.
Reservorio animal: roedores y venados.
Vector: Ixodes dammini ( garrapatas )
Estadio de larva y ninfa contienen la bacteria.
El estadio de las ninfas es el responsable de la
transmisin de la enfermedad
periodo de incubacin de 3-32 das
Sntomas relacionados con la influenza
dolor muscular y de las articulaciones
Fatiga
dolor de cabeza
fiebre, escalofros, o glndulas inflamadas.
Parlisis de los msculos faciales.
Palpitaciones
eritema migratorio
(EM).
Garrapata
de patas negras
(Ixodes capularis)

TRATAMIENTO:
doxiciclina oral 100 mg dos veces al da10-21 das
amoxicilina oral 500 mg tres veces al da14-21 das
cefuroxima oral 500 mg dos veces al daX 14-21 das

73. Varn con cefalea, rigidez de nuca, parlisis bilateral del facial y una
radiculopata perifrica dolorosa. LCR: Linfocitosis con protenas y
Glucosa normales. La meningitis muestra curso ondulante y aparece
artritis de instauracin brusca y carcter migratorio. ECG: bloqueo AV.
Microorganismo responsable de esta enfermedad:
A. Borrelia burdogferi.
B. Rikettsia coronii.
C. Rikettsia rikettsii.
D. Rikettsia prowazekii.
E. Toxoplasma gondii.

CLAVE: A

74. Florista, con antecedente de lesin cutnea producida por una espina
de rosal, desarrolla una ppula poco dolorosa en la zona de la herida,
acompaada de linfangitis. Cul es el agente MS probable?:
A. S. epidermidis
B. Clostridium tetani
C. Sporothrix schenckii
D. Streptococcos pyogenes.
E. Bartonella baciliformes

CLAVE: C

75. De las siguientes enfermedades, cual sera compatible con una zona
enrojecida e hinchada en piel con mltiples drenajes:
A. Granuloma pigeno.
B. Absceso.
C. Fornculo.
D. Lupus.
E. Carbunco

CLAVE: E

76. Con respecto a la Enfermedad de Lyme, despus de cuantos das


aparecen los sntomas luego de la picadura de una garrapata infectada:
A. 1 semana
B. 30 -60 das
C. 3 - 35 das
D. 6 meses de incubacin
E. Despus de 21 das

CLAVE: C

INMUNODEFICIENCIAS E INFECCIONES

NEOPLASIAS

Tumores rgano slido


Alteracin de mecanismos externos de defensa

Mieloma Mltiple
Alteracin de la inmunidad adaptativa humoral

Linfoma
Alteracin de la inmunidad adaptativa celular
Tratamiento inmunosupresor
Radioterapia

Leucemia linfoctica crnica


Hipogammaglobulinemia y Neutropenia transitoria
Alteracin de la inmunidad adaptativa celular: linfopenia

ALTERACIN DE LA INMUNIDAD INNATA


(I.NATURAL)
Disminucin nmero y/o trastorno de la
funcin de los neutrfilos
Sepsis por:
Escherichia coli y Enterobacterias
Pseudomonas aeruginosa
Staphylococcus aureus, S.C.N.
Estreptococos grupo viridans y Enterococcus
spp
Hongos

Deficits de factores del complemento


Sepsis por:
Neisseria spp

ALTERACIN DE LA INMUNIDAD
ADAPTATIVA HUMORAL (anticuerpos)
Alteracin en sntesis y/o funcin de las
inmunoglobulinas
Infecciones por bacterias capsuladas:
Streptococcus pneumoniae
Haemophilus influenzae

Sepsis e Infecciones aparato respiratorio

ALTERACIN DE LA INMUNIDAD
ADAPTATIVA CELULAR
Infecciones por microorganismos patgenos
intracelulares:
Bacterias
L. monocytogenes
Nocardia spp
M.tuberculosis y otras micobacterias
Hongos
P.jiroveci
Aspergillus spp
Cryptococcus spp y otros
Virus
Herpesvirus: CMV
V. Respiratorios
Parsitos: T.gondii

ESPLENECTOMA
Infecciones por bacterias capsuladas
Streptococcus pneumoniae

Haemophilus influenzae
Neisseria meningitidis
Capnocytophaga spp

Babesia spp

INFECCIONES EN
TRASPLANTE
RENAL
Entre 2-6 meses

Infeccin o enfermedad
por CMV 50%
Herpes virus:VVZ y VEB

Virus JC

INFECCIONES EN
TRASPLANTE HEPTICO
Peritonitis e infecciones
intrabdominales bacterianas
hongos
Candida spp
Aspergillus spp

Infeccin por CMV


Recidivas VHB y VHC

INFECCIONES EN TRASPLANTE DE CORAZN


Mediastinitis
Staphylococcus aureus
Staphylococcus epidermidis

Neumonas bacterianas
Candida spp
Aspergillus spp

Infecciones del S.N.C.


Toxoplasma gondii
Listeria spp y Nocardia spp

INFECCIONES EN TRASPLANTE
PULMONAR
Infecciones respiratorias
bacterianas
Mediastinitis poco frecuente
Infeccin por CMV 50 - 70%
Infecciones fngicas: Aspergilosis

77. La infeccin caracterstica de los pacientes con dficit de Ig A es


producida por:
A. Giardia lamblia
B. Estreptococo pneumoniae.
C. Clostridium.
D. Haemophilus infl uenzA.
E. Listeria monocytogenes.

CLAVE: A

78. La esplenectoma predispone a las infecciones por:


A. Nocardia
B. Brucella.
C. Estafilococo.
D. Pseudomonas.
E. Estreptococo.

CLAVE: E

79. Paciente varn con antecedente de Mieloma mltiple, acude al Hospital


por fiebre de 48 horas de evolucin, tos con expectoracin purulenta y
dolor torcico lateral. En la radiografa de trax hay un infiltrado alveolar
y en el hemograma se aprecia leucocitosis de 15.000/mm3. Cul es el
agente etiolgico MS probable?:
A. Pneumocystis jirovecii.
B. Pseudomona aeuriginosa
C. Mycobacterium tuberculosis.
D. Estreptococos pneumoniae.
E. Staphylococcus aureus.

CLAVE: D

80. Paciente varn de 30 aos adicto a drogas por va parenteral. Acude a


Urgencias por fiebre de 39C, dolor pleurtico, tos y expectoracin. En
la Radiografa de trax se observan lesiones nodulares perifricas
mltiples, alguna de ellas cavitadas. Cul sera entre las siguientes, la
prueba diagnstica que se DEBERA realizar?:
A. TAC torcica.
B. RNM pulmonar.
C. Espirometra.
D. Ecocardiografa.
E. Gammagrafa pulmonar.

CLAVE: A

BRUCELLA, NOCARDIA Y ACTINOMYCES

BRUCELOSIS
CLASIFICACION CLINICA
Brucellosis Aguda
Brucellosis subaguda
Brucellosis Cronica

< 8 sem.
8 52 sem.
>52 sem.

Medio de cultivo
Ruiz Castaeda

275

Tratamiento Brucelosis

La duracin mnima por tratamiento de


brucelosis deber ser de 6 semanas .
Los aminoglucsidos (estreptomicina
y gentamicina) administrar solamente
durante 2 semanas (14 das).

276

ACTINOMICOSIS
Flora normal
Alteracin de la
barrera mucosa

Criptas amigdalinas, encas, caries


Tracto gastrointestinal, tracto genital
femenino

Infeccin de la boca, extraccin


dental, mala higiene, trauma

Supuracin crnica Abundantes PMN y escasos


grnulos de azufre

Fistulizacin
Diseminacin por contigidad
Diseminacin hematgena

Actinomicosis: Formas clnicas


2. Torcica: por aspiracin. Forma abscesos
pulmonares
3. Abdominal: por ciruga o trauma intestinal
(Perforaciones, apendicitis)

4. Genital: portadoras de DIU


5. Diseminada: immunodeprimidos
6. Por mordedura humana

Patologa oral

Actinomyces viscosus/naeslundi
- Caries
- Inf. endodnticas
- Periimplantitis

Actinomicosis: tratamiento

Drenaje quirrgico del exudado purulento


Tratamiento antibitico durante varios meses (penicilina G a dosis altas)

Curacin: 90% de las formas cervicofaciales

NOCARDIOSIS
La familia Nocardeaceae
aerobios, Gram positivos, filamentosos, inmviles
ramificados,
acido alcohol resistentes de crecimiento lento e intracelular
N. Asteroides causa el 90% de las patologas en los humanos
ingresa al cuerpo por va inhalatoria no existiendo evidencias
de transmisin de persona a persona
En el hombre produce varios sndromes

neumona y la enfermedad diseminada


Raros: sndrome linfocutneo, el actinomicetoma, la celulitis y la
queratitis
TMP-SMX

81. Sobre BRUCELOSIS marque lo CORRECTO:


A. La recada bacteriolgica generalmente ocurre en los 3 6 meses despus de
descontinuar la terapia y usualmente es causado por resistencia antibitica.
B. Las lesiones articulares ms frecuentes son la artritis perifrica y la sacroiletis
siendo rara la espondilitis.
C. La espondilitis brucelar afecta principalmente a la columna dorsal.
D. El tratamiento de eleccin es Doxiciclina asociado a TMP-SMX durante 6
meses con lo cual se ha demostrado disminuir la tasa de recadas.
E. El compromiso ocular como uvetis, neuritis ptica y epiescleritis, son
complicaciones frecuentes en las formas subaguda o crnicas.

CLAVE: B

82. Seleccione el esquema teraputico indicado en el tratamiento de la


brucelosis humana:
A. Doxiciclina ms Cotrimoxazol por 21 das.
B. Doxiciclina ms Rifampicina por 21 das.
C. Doxiciclina ms estreptomicina por 15 das.
D. Doxiciclina ms Rifampicina por 45 das.
E. Doxiciclina ms Gentamicina por 15 das.

CLAVE: D

83. El tratamiento de eleccin en la Nocardiosis es:


A. Rifampicina + Isoniazida + Etambutol
B. Cotrimoxazol
C. Amikacina + Ceftazidima
D. Imipenem
E. Metronidazol + Gentamicina

CLAVE: B

RICKETSIAS

FIEBRES MACULOSAS O MANCHADA


A. Fiebre de las Montaas Rocosas
Bacteria que la produce : Rickettsia rickettsi
Vector: garrapatas
Reservorio : garrapatas en conejos, perros. ganado.
Sntomas- fiebre, cefalea intensa, erupcin maculopapular en
palmas de las manos y pies. Vasculitis.
Periodo de incubacin 1 semana
Evolucin sin tratamiento fallecimiento del paciente.
Tratamiento cloranfenicol y tetraciclina

FIEBRE DE LAS TRINCHERAS


Producida por la Rochilomea quintana
Vector: Pediculus humanus.
Sntomas: fiebre, cefalea, agotamiento, erupcin similar
a la rseola y vasculitis.
Hay recadas cada cinco das ( quintana )

FIEBRE BOTONOSA O
MEDITERRNEA
Rickettsia conorii
Transmitida por garrapatas
Rhipicephalus sanguineus.

Manifestaciones clnicas

Escara necrotica

Incubacin: una semana


Fiebre alta, cefalea, exantema y
artromialgias.
Escara necrtica en la zona de
inoculacin (mancha negra).
Rash maculopapular, que afecta
palmas y plantas
Infeccin benigna
Se resuelve en 1-2 semanas aun
sin tratamiento

287

FIEBRE Q
Producida por la COXIELLA BURNETTI.

No necesita vector para transmitirse.


Reservorios : garrapatas en el ganado.
MECANISMO DE INFECCION
Por inhalacin de heces secas de garrapatas de ganado bovino.
Por inhalacin de aerosoles que se producen al parir el ganado.
Puede transmitirse en leche sin pasteurizar.

Sntomas: escalofros, cefalea, malestar general, debilidad, dolores


intensos y neumona.

TIFUS EPIDEMICO
Producido por la Rickettsia prowaseki.
Vector: Piojo Pediculus humanus.
Enfermedad asociada al desaseo y suciedad.
Reservorio- hombre con piojos
Periodo de incubacin de 6 a 15 das.
Bacteremia, fiebre de 40 C, maculas,
delirio, estupor y vasculitis, sin tratamiento es
mortal

TIFUS ENDEMICO
Producido por la Rickettsia typhy (tifus murino)
Vector: Pulgas Xenopsyla cheopis.
Reservorio primario: ratas y ardillas terrestres con
pulgas
Periodo de incubacin : 1 a 2 semanas.
Sntomas: cefalea, fiebre, maculo ppulas y vasculitis.

84. Seale la afirmacin CIERTA respecto a la Fiebre Q:


A. Se transmite por inhalacin de partculas contaminadas.
B. Los hemocultivos son positivos en la fase inicial.
C. La mancha negra se observa en el 60% de los casos.
D. El exantema suele afectar palmas y plantas.
E. El tratamiento de eleccin es la Gentamicina.

CLAVE: A

85. Acude a su guardia en la clnica un turista varn de 30 aos con fiebre y


cefalea hace 48 horas. En el examen fsico, adems de fiebre de 38,5C se
observa un exantema maculo-papuloso generalizado que afecta a palmas y
plantas. Tambin se encuentra una pequea lesin costrosa negruzca en
una pierna. Cul de los siguientes es el diagnstico MS probable?:
A. Fiebre botonosa.
B. Les secundaria.
C. Tifus murino.
D. Mononucleosis infecciosa.
E. Sndrome de Stevens-Johnson.

CLAVE: A

BARTONELLA

BARTONELOSIS
El Agente Causal
El gnero Bartonella, perteneciente a la
familia Bartonellaceae, incluye
actualmente 14 especies.
De ellas, se han descrito siete especies
patgenas para el ser humano:
B. bacilliformis, B. quintana, B.vinsonii,
B. elizabethae, B. henselae, B.
clarridgeiae y B. grahamii.

El Vector
La Lutzomyia verrucarum.
La hembra del mosquito es hematfaga,
con mayor actividad antropoflica entre las
18 y 19 h.

Cuadro Clnico
El cuadro clnico puede variar desde una
forma subclnica hasta una presentacin
aguda y fulminante.
El periodo de incubacin tiene un
promedio de 61 das (10 - 210)

La enfermedad es bifsica, con una fase


anmica y una eruptiva separadas por un
periodo intermedio asintomtico.
La fase aguda se caracteriza por fiebre,
palidez, ictericia, hepatoesplenomegalia y
adenopatas.
La fase eruptiva por la aparicin de
hemangiomas.

La letalidad es mayor en caso de brotes


(hasta 88%)
294

Bartonelosis
DIAGNSTICO

AGUDA O ANEMICA
Adultos :
Ciprofloxacino 500 mg c/12h. VO por 14 das

Alternativas
:
El frotis de sangre perifrica es el mtodo ms
Amoxicilina/Ac.clavulnico
prctico y barato para diagnosticar la
Cotrimoxazol
enfermedad. La sensibilidad es baja(36%) pero
Nios :
la especificidad es alta (96%).
El aislamiento en cultivos es ms sensible,
pero requiere mayor infraestructura, existe
riesgo de contaminacin y presenta un
periodo de incubacin prolongado.

Amoxicilina /Acido clavulnico

Alternativas :
Ciprofloxacino
Cloranfenicol
Gestante :
Amoxicilina/Ac. Clavulnico

Alternativas :
Cloranfenicol
Cotrimoxazol

ERUPTIVA
Droga de eleccin : Azitromicina x 7d.
Gestantes : 1 gr. Semanal VO por 3
semanas
295

BARTONELOSIS
Infecciones Oportunistas
Salmonella Typhi y non-typhi
Reactivacion toxoplasma, reactivacion TBC , reactivacin Hep. B
Histoplasmosis diseminada,
sepsis debido a Shigella, Staphylococcus, Enterobacter
Neumocystosis
Typhus,
Leptospirosis
Influenza

296

86. Cul de las siguientes infecciones es la MS frecuente en pacientes


con Bartonellosis?:
A. Neumona por Pneumocystis.
B. Bacteriemia neumoccica.
C. Bacteriemia por Salmonella.
D. Erupcin por Herpes zoster.
E. Neumona por H. influenza.

CLAVE: C

87. Paciente de 40 aos de edad, viene directamente desde Abancay al servicio


de Emergencia. Se observa paciente soporoso, fiebre de 40 C, severa
palidez de piel y mucosas, leve ictericia y hepatoesplenomegalia. Lab: Hb:
4g%; leucocitos: 18.000 mm3, reticulocitos: 15%; Test de Coombs: (-); Test
de Ham: (-); Gota gruesa: (-). La posibilidad diagnostica MS probable, sera:
A. Infeccin por Clostridium perfringens.
B. Anemia hemoltica autoinmune.
C. Hemoglobinuria paroxstica nocturna.
D. Malaria por P. falciparum.
E. Bartonellosis.

CLAVE: E

88. Paciente de 10 aos, procede de la sierra central del departamento de


Ancash, llega con un tiempo de enfermedad de 12 das, presenta
decaimiento general, fiebre alta, escalofrios, palidez progresiva y
visceromegalia. El diagnostico PROBABLE es:
A. Bartonellosis.
B. Brucelosis.
C. Dengue.
D. Fiebre Amarilla.
E. Malaria.

CLAVE: A

89. Adems de la infeccin por Salmonella Qu otras infecciones


secundarias se acompaan frecuente a la fiebre de la Oroya?:
A. Sarna costrosa-encefalitis viral.
B. Hepatitis viral-Influenza.
C. Paludismo-TBC.
D. Micetoma-leishmaniosis.
E. Criptococosis-dengue.

CLAVE: B

90. De acuerdo al Manual del MINSA, el medicamento de eleccin para el


tratamiento de la Bartonellosis aguda en adolescentes mayores de 14
aos y adultos (> 45 kg) es:
A. Ciprofloxacina.
B. Penicilina procanica.
C. Estreptomicina.
D. Tetraciclinas.
E. Cloramfenicol

CLAVE: A

ENFERMEDADES POR VIRUS

MONONUCLEOSIS
INFECCIOSA
Producido por virus EBSTEIN BARR
Enfermedad aguda caracterizada por FARINGITIS,
FIEBRE y LINFADENOPATA, aparicin transitoria
de Ac heterfilos, leucocitosis MN (50%) y ms de
10% de linfocitos atpicos.
Periodo de incubacion: 4 6 semanas
La EXPRESIN CLNICA vara de acuerdo a la
edad:
Nios pequeos: Usualmente asintomtica y
Ac H (-)
Adolescentes: mayor frecuencia de rash,
neutropenia y neumona en relacin a los
adultos

EBSTEIN BARR
COMPLICACIONES

304

DENGUE
Reservorio El Hombre
Modo de Transmisin: (Aedes aegypti)
Tiempo de Incubacin:
3 a 14 das
Periodo de Transmisibilidad: Hasta 7
das
Susceptibilidad:
Universal
Familia: Flavivirus
Cuatro serotipos

Definicin de caso probable


Vive/viaj a zona endmica y
fiebre menor de 7 das, ms
dos de los siguientes :
Anorexia/nuseas
Mialgias/artralgias
Exantema
Cefalea/dolor retro-ocular
Petequias o prueba del lazo
positiva
Leucopenia
Cualquier signo de alarma

305

CURSO CLNICO DEL DENGUE


1

Das de enfermedad

40

Temperatura

Deshidratacin

Eventos clnicos
potenciales

Cambios en el
laboratorio

Hematocrito
Viremia

Serologa y virologa

Curso clnico
Fase febril
de la enfermedad:

S
I
G
N
O
S
D
E

Shock
Hemorragia
Dao de
rganos

A
L
A
R
M
A

Fase critica

10

Reabsorcin de lquidos

Plaquetas

IgM / IgG

Fase de recuperacin

Adapted from WCL Yip, 1980 by Hung NT, Lum LCS, Tan LH

306

307

FIEBRE AMARILLA
Fiebre hemorrgica viral,
potencialmente mortal.
Familia Flaviviridae (Arbovirus
del grupo B)
Transmisin al ser humano
Gnero Aedes (FAU)
Gneros Haemagogus y Sabethes (FAS)

Incubacin
Dos a seis das
Tasa de letalidad para las formas
clnicas graves oscila entre 50 a
80%

308

LABORATORIO
Pruebas serolgicas
IgM por ELISA.
IgM + IgG por IFI.

Aislamiento viral:
til en casos de sospecha de Fiebre
Amarilla en pacientes virmicos
(consulta temprana).

PCR

91. El diagnstico clnico de Mononucleosis infecciosa por virus EpsteinBarr, incluye fiebre y:
A. Faringitis, adenomegalia
B. Artralgias.
C. Faringitis, visceromegalia.
D. Artralgias, visceromegalia.
E. Adenomegalia, visceromegalia.

CLAVE: A

92. Paciente de 20 aos, dolor de garganta, fiebre. Al examen: ictericia


subclnica, adenopatas cervicales y bazo palpable. Lab: linfocitosis,
clulas linfoides activadas, aumento leve de TGO y TGP. El
diagnstico MS probable es:
A. Virus de Epstein Barr.
B. Hepatitis A subaguda.
C. CMV.
D. Hepatitis B subclnica.
E. Infeccin por Herpes tipo 1.

CLAVE: A

93. En el diagnstico de dengue hemorrgico, Qu aspecto es el MS


importante?:
A. Trombocitopenia
B. Shock hipovolemico con hemoconcentracin.
C. Shock hemorrgico.
D. Antecedente de haber presentado dengue clsico. (reinfeccion)
E. Procede de una zona endmica para dengue

CLAVE: D

94. Paciente varn de 19 aos que presenta 6 das de fiebre asociados a dolor
muscular generalizado y dolor ocular. Al quinto da present rash macular
generalizado y dolor abdominal. Lab: Anemia leve (Hto: 32%), leucopenia
(3500 cel./mL), plaquetopenia (80,000 cel/mL), no elevacin de
transaminasas. Gota Gruesa (-). Diagnstico. MS probable segn el MINSA
es:
A. Dengue severo.
B. Malaria.
C. Dengue clsico.
D. Dengue con signos de alarma.
E. Dengue sin signos de alarma.

CLAVE: D

95. Paciente varn de 25 aos natural y procedente de Pucallpa que presenta 5


das de fiebre, malestar general, inyeccin conjuntival e ictericia. Dos das
antes de ingreso present hematemesis y hemoptisis. El examen fsico
revel fiebre, bradicardia, hipotensin, lesiones purpuricas y trastorno del
sensorio. Lab: anemia, leucocitosis, hipertransaminasemia,
hiperbilirrubinemia directa y proteinuria. Cul es el diagnstico MS
probable?:
A. Dengue.
B. Leptospira.
C. Hanta virus.
D. Fiebre amarilla.
E. Malaria.

CLAVE: D

96. Los serotipos de Dengue, se reconocen por la variacin de la protena:


A. C2
B. M4
C C3
D. E2
E. E4

CLAVE: E

INFECCION POR VIH

ESTADOS VIH (CLASIFICACIN CDC 1993)


Clnico
Inmune
CD4

>500

A1

B1

C1

200 - 500

A2

B2

C2

< 200

A3

B3

C3

CATEGORIA CLINICA C CON INFECCION POR VIH


Candidiasis de bronquios, trquea o pulmones o esofgica
Cncer cervical invasivo
Coccidiodomicosis, diseminada o extrapulmonar
Criptococosis extropulmonar
Criptosporidiosis intestinal crnica (> de un mes de duracin)
Complejo Mycobacterium avium intracelullare
Enfermedad por citomegalovirus (no incluye heptica, esplnica ni
linftica)
Encefalopatia relacionada al VIH
Herpes simplex, lcera(s) crnica(s); o bronquitis, neumonitis o esofagtis
Histoplasmosis, diseminada o extrapulmonar
Isosporiasis intestinal crnica (> de un mes de duracin)
Linfoma de BurKitt, inmunoblstico o primario cerebral
Leucoencefalopata Multifocal Progresiva
Mycobacteriium tuberculosis, de cualquier localizacin (pulmonar o
extrapulmonar)
Neumona por Pneumocystis carinii
Sarcoma de Kaposi
Sndrome de consumo por VIH

CLASIFICACION ESTADIOS VIH SEGN OMS 2007

TRATAMIENTO ANTIRRETROVIRAL

Se iniciar TARGA con los siguientes criterios:


Toda persona con infeccin por el VIH que presente sntomas relacionados a
inmunosupresin (Estadios Clnicos 2, 3 y 4 de la clasificacin de OMS 2007).
Toda persona con CD4 500 clulas/mm3, independientemente de la presencia de
sntomas.
Toda persona que presente alguna de las siguientes condiciones:
a. Nefropata relacionada al VIH.
b. Deterioro Neuro-cognitivo asociado a VIH.
c. Neoplasias no relacionadas al VIH que necesiten quimio o radioterapia
d. Co infeccin con Hepatitis B crnica que requiera tratamiento.
e. Co- infeccin Hepatitis C crnica que requiera tratamiento.
f. Enfermedades autoinmunitarias relacionadas a VIH.
g. Edad mayor de 55 aos.
h. Gestacin.
i. Paciente con Infeccin VIH y con pareja serodiscordante.
Toda persona con infeccin aguda por el VIH, que presente sntomas relacionados a
infeccin aguda.
NORMA TCNICA DE SALUD DE ATENCIN INTEGRAL DEL ADULTO CON INFECCIN POR EL VIRUS DE LA INMUNODEFICIENCIA HUMANA (VIH)

Esquemas de tratamiento

6.2.3.5 El esquema de tratamiento antirretroviral recomendado para pacientes que por primera
vez inician el tratamiento, est basado en la combinacin de dos medicamentos inhibidores
nuclesidos/nucletidos de la transcriptasa reversa (2 INTR) ms un medicamento inhibidor no
nuclesido de la transcriptasa reversa (1 INNTR).
6.2.3.6 La combinacin de antirretrovirales para el esquema de primera lnea es

Tenofovir 300 mg/Emtricitabina 200 mg/Efavirenz 600 mg,


en dosis fija combinada, siendo la dosis en adultos de una
tableta cada 24 horas (antes de acostarse).

NORMA TCNICA DE SALUD DE ATENCIN INTEGRAL DEL ADULTO CON INFECCIN POR EL VIRUS DE LA INMUNODEFICIENCIA HUMANA (VIH)

Curso de la Infeccin y la Enfermedad por VIH en adultos:


puntos de decisin
CD4

1000

Sind. Retroviral Agudo (25-70%)

900

Evaluacin inicial
Profilaxis contra TBC

600
550
500

Profilaxis contra PC
y Toxoplasma (TMP-SMX)

450
250
200

Profilaxis contra MAC


Profilaxix anti-mictica

150
100

Profilaxis contra CMV

50
0

3 6 9 12

Semanas

1 2 3 4 5 6 7 8 9 10 11

Aos

CULES SON LOS EFECTOS ADVERSOS DEL TARGA?

Escenario 1
Gestante diagnosticada
por primera vez
TDF
TDF ++ 3TC
3TC ++ EFV
EFV
desde
desde la
la semana
semana 14
14

Escenario 2
Gestante que ha estado
recibiendo TARGA
Continuar con TARGA en el embarazo y
despus del parto

Escenario 3
Gestante diagnosticada
por primera vez en el
momento del parto

no Sustituir EFV si es que esta recibiendo

TDF + 3TC + EFV


mas AZT

AZT por 4 semanas

AZT por 4 semanas

AZT por 6 semanas

Si madre recibi TARGA menos


de 4 sem. Entonces AZT por 6
sem.

Si madre recibi TARGA menos


de 4 sem. Entonces AZT por 6
sem.

NVP al 1, 3 y 7 dia

Suspender lactancia materna

Suspender lactancia materna

Suspender lactancia materna

PREVENCION DE TRANSMISIN

97. Paciente con diagnstico VIH positivo con CD4: 50, que acude febril
desde hace 3 semanas, con infiltrado intersticial difuso en ambos
campos pulmonares. PPD (-), Giemsa en esputo (-) y BK en esputo
(+++). Se sabe que el paciente requiere tratamiento antituberculoso, as
como terapia antirretroviral (ARV) por lo que Usted decide:
A. Iniciar terapia ARV antes que antiTBC.
B. Iniciar antiTBC junto con ARV.
C. Iniciar antiTBC y luego ARV.
D. Finalizar antiTBC y luego dar ARV.
E. Slo dar antiTBC.

CLAVE: C

98. Paciente mujer de 28 aos recientemente diagnosticada de infeccin


por VIH durante la gestacin. Actualmente tiene 16 semanas de
gestacin por fecha de ltima regla. Usted decide iniciar antirretrovirales
para disminuir la transmisin materna perinatal. Cul de los siguientes
regmenes elegira?:
A. Zidovudina.
B. Esperara hasta que cumpla 20 semanas de gestacin.
C. Didanosina + estavudina.
D. Zidovudina + lamivudina + lopinavir/ritonavir.
E. Zidovudina + lamivudina + Efavirenz.

CLAVE: E

99. El criterio que mejor establece la necesidad de iniciar terapia


antirretroviral es:
A. Recuento de CD menor a 200 clulas/mm3.
B. Carga viral superior a 30000 copias/ml (RT-PCR).
C. Recuento de CD4 inferior a 350 clulas/mm3.
D. Carga viral superior a 10000 copias/ml (RT-PCR).
E. Recuente de CD4 menor a 400 clulas/mm3

CLAVE: A

100.Cul de las siguientes complicaciones neurolgicas del SIDA es


espontneamente reversible?:
A. Demencia
B. Leucoencefalopata multifocal progresiva.
C. Sndrome de Guillain-Barr.
D. Mielopalia vacuolar.
E. Toxoplasmosis cerebral.

CLAVE: C

101.Paciente infectado por VIH, con linfadenopatia generalizada persistente


y cifras de CD4 de 300/mm3, corresponde en el sistema de clasificacin
clnica:
A. A1
B. A2
C. A3
D. B2
E. B3

CLAVE: B

INFECCIONES POR HONGOS

CANDIDIASIS
Es una levadura, un gram positivo se reproduce por gemacin.
Existen mas de 200 especies
Solo 10 son patgenos: C .albicans, C. Tropicalis, C Krusei,
C.parapsilosis, C.guillermondii, C.Glabrata.

Infeccin local:
Cutanea
Candidiasis orofaringe
Esofagitis
Vulvovaginitis
Candidiasis mucuocutanea
crnica
Infecciones Invasivas focales
Infeccin del tracto urinario
Peritonitis
Endoftalmitis
Infeccin articular
Meningitis
Candidiasis hepatoesplnica

Tratamiento:
Topico: Terbinafina, clotrimazol, Ketoconazol
Sistemico: fluconazol
Cuadros severos: anfotericina B

PARACOCCIDIOIDOMICOSIS O
BLASTOMICOSIS SUDAMERICANA
Infeccion fungica sistemica de mayor
prevalencia en America Latina
Etiologia: Paracoccidioides brasiliensis
Hongo dimorfico
INGRESA POR INHALACION
FORMAS CLINICAS
Pulmonar

Forma pulmonar aguda


Forma pulmonar cronica

Linfonodular
Mucocutanea
Mixta

TRATAMIENTO

ANFOTERICIN B 0.5 1 mg/kg/da hasta


completar 1.5 a 2 gr luego Itraconazol

HISTOPLASMOSIS
Histoplasma capsulatum
Es un hongo dimrfico y geoflico
Formas
diseminadas
en
inmunodeprimidos (VIH, corticoides)
No hay transmisin interhumana ni
de animales al hombre.
Inhalacin
de
microconidias,
penetran al alveolo pulmonar,
fagocitados por macrfagos, se
transforman en levaduras gemantes
Reaccin granulomatosa epiteliode,
cel. Gigantes, necrosis caseosa

1. Histoplasmosis en el hospedero normal


a) Primoinfeccin asintomtica 99%
b) Infeccin Pulmonar Aguda
2. Histoplasmosis en pacientes con EPOC
a) Histoplasmosis Pulmonar Crnica Cavitaria
3. Histoplasmosis en el inmunocomprometido
a) Histoplasmosis diseminada aguda
b) Histoplasmosis diseminada subaguda
c) Histoplasmosis diseminada crnica
4. Enfermedad mediada inmunolgicamente,

TRATAMIENTO

ANFOTERICIN B 0.5 1 mg/kg/da hasta completar 1.5


a 2 gr luego Itraconazol

MUCORMICOSIS
Varias enfermedades producidas por
hongos Mucorales.
Rhizopus, Rhizomucor, Cunninghamella
y Absidia.
Inhalacion de conidias es rutinaria por
hospederos inmunocompetentes.
Baja virulencia
Hospedero inmunocomprometido

Ingresa por inhalacion, o piel lesionada.


Formas clinicas:
Rinocerebral
Pulmonar
Cutanea
Gastrointestinal
SNC
Miscelanea
Diabetes mellitus:
rinocerebral
Neutropenia:
rinocerebral y pulmonar
Desnutricion:
gastrointestinal
Leucemia aguda:diseminada

El tratamiento mdico estndar contina siendo la


anfotericina B

Pneumocystis jiroveci
VIH/SIDA
Cncer
Trasplante de
rganos slidos o
mdula sea
Corticosteroides o
inmunosupresore
s
EPOC?
Fiebre
Tos, a menudo leve y
seca
Disnea
Taquipnea
Taquicardia
Cianosis.

No tiene ergosterol
Metodos diagnostico:
Estudio de esputo en
Gomori Grocott
Inmunofluorescencia

Tratamiento: sulfametoxazol/trimetropim, en
alrgicos a sulfas, Primaquina/clindamicina x 21
dias

prednisona

102.Paciente diabtica 32 aos obesa, piel roja y dolorosa en sus pliegues


abdominales. Material blanco cremoso en la base del pliegue; eritematoso
por debajo y se extiende ms all del material cremoso. Microscopio:
estructuras ovales de gemacin (3x6um) mezcladas con formas ms
elongadas de gemacin. El agente etiolgico MS PROBABLE es:
A. Aspergillus.
B. Candida albicans.
C. Epidermophyton floccosum.
D. Microsporum.
E. Sporothrix schenckii.

CLAVE: B

103.La afeccin de senos paranasales en inmunodeprimidos es tpica de


uno de los siguientes hongos:
A. Cndida.
B. Histoplasma.
C. Mucor.
D. Criptococo.
E. Aspergillus.

CLAVE: C

104.Paciente varn de 24 aos procedente de La Merced Junn que presenta 4


meses de tos con escasa expectoracin, dificultad respiratoria progresiva y
cada de piezas dentarias. Rx trax: infiltrado intersticial bilateral con
retracciones y cavidades en ambos campos pulmonares. Ziehl-Neelsen de
esputo (-) y el KOH mostro mltiples estructuras compatibles con levaduras
con gemacin satlite en forma de timn de barco. Cul de los siguientes
es el diagnstico MS probable?:
A. Paracoccidiomicosis.
B. Histoplasmosis.
C. Blastomicosis.
D. Aspergillosis.
E. Zigomicosis

CLAVE: A

INFECCIONES POR PARASITOS

Criterios severidad OMS

MALARIA

Malaria cerebral (coma)


Anemia severa (Hto<15%)
Insuficiencia renal
(creatinina > 3 mg%)
Edema agudo de pulmn
Hipoglicemia (glicemia <
40 mg%)
Shock (PAS < 70 mmHg)

Anopheles
pseudopunctipennis

Especies que afectan al


humano:
Plasmodium vivax
Plasmodium falciparum
Plasmodium malariae
Plasmodium ovale
Plasmodium knowlesi

Cuadro clnico

Diagnstico

Fiebre
Escalofros
Visceromegalia
Anemia

Examen directo

Frotis / gota
gruesa

339

Caractersticas de Infeccin por Plasmodium de acuerdo a


especies

Campuzano G , Blair S ( 2010).Malaria : Consideraciones para su diagnstico. Medicina y Laboratorio.


Vol 16.

MALARIA
ESQUEMAS DE TRATAMIENTO para Plasmodium vivax
MEDICAMENTO

Plasmodium vvax
para Adultos

DAS

N
DAS

CLOROQUINA 250mg
PRIMAQUINA 15 mg

7
7,5 mg

PERODO POST
PARTO

Perodo desde la aparicin de la enfermedad hasta el parto


Desde el momento del Diagnstico hasta el parto
MEDICAMENTO

Plasmodium vvax
para Gestantes

SEMANA 1

PRIMER MES POST


PARTO
1

Das
1

CLOROQUINA

Tab 250 mg
(150mg base)

DAS

2 Tab Cloroquina semanal hasta el parto

PRIMAQUINA
Tab 15 mg

2 2 2 2 2 2 2

NO ADMINISTRAR PRIMAQUINA A GESTANTES

341

MALARIA
ESQUEMAS DE TRATAMIENTO para Plasmodium malariae

Tratamiento P. falciparum

343
RESOLUCIN MINISTERIAL N 116-2015/MINSA

Tratamiento P. falciparum

LEISHMANIASIS
TEGUMENTARIA

Lesiones en reas expuestas del cuerpo:


1. lceras,
2. Tejido granulomatoso grueso,
3. Bordes elevados,
4. no cura espontnea en 4 semanas

CICLO BIOLGICO
Husped Mamfero
Ciclo Enzootico
Promastigote

Reservorio
Mamfero

Lutzomyias
Amastigote

345

CLASIFICACION CLINICA

DIAGNSTICO LABORATORIAL
Inmunolgico: leishmanina (80-92%)
Mtodos Visuales: (21% - 72%)
Frotis por impresin
Histopatologa (bajo)

Mtodos parasitolgicos
(aislamiento):
Cultivos in vitro (57% - 85%)

PCR: 92-94%

Son las drogas de eleccin:


Stibogluconato de Sodio (Pentostam)
Antimoniato de Meglumine (Glucantime )
Anfotericina B (en caso de resistencia)
Praziquantrel
azoles

TOXOPLASMOSIS
Taxonoma y Biologa
-Toxoplasma gondii
-Tipo: Apicomplexa.
-Subclase: Coccidia.

Formas Evolutivas
-Taquizoito.(Pseudoquiste y formas libres)
-Bradizoito. (Quistes tisulares)
-Esporozoito. (Ooquiste)

347

TOXOPLASMOSIS
Transmisin por ingestin:
Esporozoitos en ooquistes esporulados

Bradizoitos en quistes tisulares presentes en


carnes crudas o poco cocidas.
Transmisin congnita:
Taquizoitos y parasitemia en caso de
infeccin aguda materna
Otras Vas: Transfusional, transplantes, etc

TOXOPLASMOSIS
Husped
inmunocompetente

Husped
inmunodeprimido

Forma asintomtica (80 a 90%)


Toxoplasmosis ganglionar.
Toxoplasmosis ocular
Otras

Reactivaciones
Siempre formas graves
Toxoplasmosis cerebral ms frecuente
Toxoplasmosis pulmonar
Toxoplasmosis diseminadas
Otras

Toxoplasmosis Ocular

Representa 3 a 10% de todas las uveitis


Representa 10 a 60% de las uveitis
posteriores
Alto grado de recurrencia (13 a 39%)

Toxoplasmosis Congnita

Consecutiva a primoinfeccin materna


durante la gestacin.
El riesgo de presentar sntomas al
nacimiento es mayor en caso de infeccin
en etapas fetales precoces.

349

TOXOPLASMOSIS

tratamiento

El rgimen de eleccin consiste en


PIRIMETAMINA (100-200 mg de dosis inicial, despus 50-100
mg/dia)
SULFADIAZINA 4-8 g/da por va oral mantenido durante al
menos 6 semanas acido folnico 10 mg/da

350

TRIPANOSOMIASIS O
ENFERMEDAD DE CHAGAS
La
Enfermedad de Chagas o Trypanosomiasis
Americana es una parasitosis causada por el
Trypanosoma cruzi y transmitida por insectos de la
familia Reduviidae.

19 ESPECIES DE LOS TRES PRINCIPALES


GNEROS DE TRIATOMINOS : Triatoma,
Panstrongylus y Rhodnius SE HAN HALLADO EN
EL PAS.
En el Per, es endmica en el Sur-Occidental:
Arequipa, Moquegua, Madre de Dios y en la zona
Nor-Oriental: Cajamarca y Amazonas.

ENFERMEDAD DE CHAGAS
FORMAS CLINICAS

CONGENITA

PREMATURO
FIEBRE
HEPATOMEGALIA
ESPLENOMEGALIA
ADENOPATIAS
MIOCARDITIS

AGUDA

CHAGOMA
FIEBRE
HEPATO Y
ESPLENOMEGALIA
ADENOPATIAS
MIOCARDITIS

CRONICA

DESPUES DE 10 AOS
MIOCARDITIS PROG.
DILATACION IRREVESIBLE
VISCERAS HUECAS
MEGAESOFAGO
MEGACOLON
MEGA URETER
OTRAS VISCERAS HUECAS

DIAGNOSTICO LABORATORIAL
EXAMENES
PARASITOLOGICOS
(DIRECTOS)
-Examen en Fresco.
-Concentracin de Strout.
-Frotis y Gota Gruesa CON GIEMSA
-Hemocultivo.
-Xenodiagnostico.
-PCR.
EXAMENES SEROLOGICOS (INDIRECTOS).
-IFI
-ELISA.
XENODIAGNOSTICO

TRATAMIENTO
-El tratamiento es posible en las formas aguda y congnita, en las
cuales, la abundancia de parsitos hacen posible la curacin
parasitolgica, lo que no ocurre en la etapa crnica donde la
parasitemia es escasa.

-NIFURTIMOX
-BENZNIDAZOLE
-En Insuficiencia Cardiaca o Mega formaciones:
REPARADORA.

CIRUGA

105.La denominada terciana benigna, con periodo de incubacin de 14


das, transmisibilidad entre uno a tres aos y que se acompaa de
fiebre irregular, escalofros, diaforesis, cefalea, astenia, ictericia, es
producida por Plasmodium:
A. Falciparum
B. Malarie
C. Ovale
D. Vivax
E. Tropicalis

CLAVE: D

106. Paciente varn de 20 aos, consulta por cuadro de fiebre no cuantificada cada 3
das, cefalea frontal, escalofros de predominio vespertino. Examen fsico T: 40 C,
FC 90, FR 20, PA 110/70 mmHg, peso 60 kilos. Dolor abdominal y
hepatoesplenomegalia. Lab: Hto 27.9%, Hb: 9.3 g/dl, Frotis de sangre perifrica y
Gota Gruesa: (+) con presencia de Gametocitos de Malaria (++), de acuerdo a la
historia del paciente se puede presumir que se trata de la especie:
A. Flavivirus, transmitida por el Aedes aegypti.
B. Leishmania donovani, transmitida por Lutzomia.
C. Tripanosoma cruzi, transmitida por el Triatoma.
D. Plasmodium vivax, transmitida por el Anopheles.
E. Toxoplasma gondii, transmitido por Artrpodos.

CLAVE: D

107.La prueba de oro en leishmaniosis cutnea andina es:


A. Prueba de ELISA
B. Hallazgo de amastigotes en el frotis
C. Anlisis de isoenzimas
D. Buena respuesta al tratamiento con Glucantime
E. Reaccin intradrmica de Montenegro positiva

CLAVE: B

108.Paciente varn que procede de un medio rural costero con mal


saneamiento ambiental, con antecedente de ingesta de verduras
crudas. Presenta sndrome disentrico. Cul es agente causal MS
probable del cuadro clnico?:
A. Sarcocystis aucheniae.
B. Balantidium coli.
C. Trichocephalus dispar.
D. Giardia lamblia.
E. Entamoeba histolytica.

CLAVE: E

INSECTOS, ARTROPODOS Y OFIDIOS


VENENOSOS

LOXOCELISMO
Loxosceles laeta y Loxosceles rufipes son las especies ms
frecuentes en el Per
Loxosceles es una especie solitaria, de hbito intradomiciliario,
nocturno
El componente ms importante es la Esfingomielinasa - D, con
actividad dermonecrtica y hemoltica
Existen dos formas clnicas claramente definidas : Loxocelismo
cutneo y Loxocelismo cutneo-visceral

La mayor cantidad de casos (70-90%) corresponden a casos


de loxocelismo cutneo
Tratamiento: Suero

antiloxosclico monovalente

(INS)
360

LATRODECTISMO
Latrodectus Mactans
Es predominantemente rural, con un hbitat
extradomiciliario, y a veces en jardines.
El veneno de la araa tiene como componente
activo la alfalatroinsectotoxina
El veneno es neurotxico, produce un fuerte
dolor en el lugar de la mordedura, sudoracin
intensa, salivacin y nuseas; das despus
eczema, en casos graves, coma.
Medidas especificas: el suero antilatrodectus

361

109.Adolescente de 12 aos de edad, procedente de un valle costero de la


regin suroccidental de Per. Presenta al examen: edema bipalpebral
unilateral, adenopata preauricular, hiperemia de la conjuntiva y
dacriocistitis. Cul es el diagnstico ms probable?:
A. Loxocelismo
B. Enfermedad de Chagas crnica
C. Enfermedad de Chagas aguda
D. Latrodectismo
E. Escorpionismo

CLAVE: C

110.En caso de mordedura recuente por araa viuda negra, Cul es la


medida ms especfica e inmediata a administrar?:
A. Corticoides.
B. Suero antiLatrodectus.
C. Carbonato de calcio.
D. Antibiticos.
E. Clorfenamina.

CLAVE: B

ematos88@hotmail.com

S-ar putea să vă placă și